Sunteți pe pagina 1din 77

Q2

A childs knee at the age of 3 years would normally show this alignment
Varus of 20 degrees
Varus of 10 degrees
Neutral alignment
Valgus of 10 degrees
Valgus of more than 20 degrees

References

During development children's tibio-femoral alignment changes during early years. At birth it
is 10-15 degrees of varus, which remodels to neutral at about 24 months age and then
becomes 10 degrees of valgus at the age of 3 years. Over the next 4 years it gradually goes
to normal alignment of 7 degrees. Salenius P, Vankka E. The development of the tibio-
femoral angle in children. JBJS Am 1975, vol 57, pp 259 – 261.

Question 3
The following are contra-indications to hip spica immobilization of paediatric femoral
fractures:
Greenstick fracture of the midshaft femur
Closed fracture of femur distal metaphysis
Visceral trauma
Bilateral femoral diaphyseal fractures
2cm shortening

References

Contra-indications for hip spica are: unacceptable shortening or angulation after closed
manipulation, open fractures, thoracic or intra-abdominal trauma, very large or obese children
and children over 6 years old (due to difficulty with hygiene, toileting and general care of
child).
http://www.wheelessonline.com/ortho/hip_spica_cast
Improved treatment of femoral shaft fractures in children. The "pontoon" 90-90 spica cast.
Miller-ME; Bramlett-KW; Kissell-EU; Niemann-KM. Clin-Orthop. 1987 Jun(219): 140-6

Question 4
Which of the following factors has NOT been implicated in an increased incidence of
non- union of fractures?
Low Molecular Weight Heparin
Non Steroidal Anti Inflammatory Drugs
Statins
Age
Corticosteroids
References

Gaston MS, Simpson AHRW. Inhibition of fracture healing. JBJS 2007; M. S. Gaston, 89-B:
1553-1560.

Question 5
A 35 year cricket player presents with pain over the lateral aspect of the elbow
approximately 5mm distal and anterior to the midpoint of the lateral epicondyle. On
clinical examination pain is exacerbated by resisted wrist dorsiflexion. Which one
statement about this condition is true?
Occurs most frequently in athletes.
Operative treatment is required in almost 20% cases.
It is initiated as a inflammatory process of the origin of the ECRB or ocasionally the ECRL,
EDC or ECU.
Microscopic evaluation demonstrates angiofibroblastic hyperplasia.
In those patients who need surgery, Maximal improvement is only seen after 6 months.

References

1) Campbells operative orthopaedics.Vol 3 Tenth Edition. Chapter 44, Shoulder and Elbow
injuries; pp. 2361-2363.
2)Millers Review of Orthopaedics.Fourth Edition.Section 5, Elbow Injuries; p.244.
The diagnosis is Tennis elbow. It happens more commonly in Non Athletes with a peak age
Incidence of 50. Non operative treatment is successful in 95 % cases. Pathologically it
involves microtears of the origins of the ECRB mainly but it can also involve ECRL, EDC and
ECU. Pathologically it is not an inflammatory process but in fact consists of angiofibroblastic
hyperplasia. Many surgical options are used in recalcitrant cases but improvement after
surgery in carefully selected cases is seen within the first 3-4 months.

Question 6
In the anterior approach to cervical spine, the platysma muscle needs to be split as
part of the surgical approach. It is supplied by which nerve ?
C2 spinal nerve
The spinal part of the accesory nerve
The superior laryngeal nerve
The facial nerve
C1 spinal nerve

References

The platysma is a superficial muscle that overlaps the sternocleidomastoid.

It is a broad sheet arising from the fascia covering the upper parts of the pectoralis major and
deltoid; its fibers cross the clavicle, and proceed obliquely upward and medially along the side
of the neck.

The anterior fibers interlace, below and behind the symphysis menti, with the fibers of the
muscle of the opposite side; the posterior fibers cross the mandible, some being inserted into
the bone below the oblique line, others into the skin and subcutaneous tissue of the lower part
of the face. Many of these fibers blend with the muscles about the angle and lower part of the
mouth.

Sometimes fibers can be traced to the zygomaticus, or to the margin of the orbicularis oculi.
Beneath the platysma, the external jugular vein descends from the angle of the mandible to
the clavicle.

The platysma is supplied by the cervical branch of the facial nerve.

When the entire platysma is in action it produces a slight wrinkling of the surface of the skin of
the neck in an oblique direction. Its anterior portion, the thickest part of the muscle, depresses
the lower jaw; it also serves to draw down the lower lip and angle of the mouth in the
expression of melancholy

Question 7
Tranexamic acid has been shown to be useful in reducing postoperative blood loss in
some situations. The mechanism of action of tranexamic acid is
As a peripheral vasoconstrictor
By enhancing platelet activation
By catalysing the conversion of fibrinogen to fibrin
By inhibition of plasminogen activation and fibrinolysis
By accelerating the production of clotting factors by the liver

References

Tranexamic acid is a man-made derivative of the naturally occurring amino acid lysine and
works to inhibit, on a molecular basis, the break down of blood clots. Retrospective case
control study of hip replacement has shown significant reduction in requirement of transfusion
with use of Tranexamic acid. Phillips SJ et al. Does salvage and tranexamic acid reduce the
need for blood transfusion in revision hip surgery? JBJS 2006; 88-B: 1141-2.

Question 8
Contraindication for Gallows traction for fractures of the femur include all of the below
EXCEPT
Open fracture
Skin contusions
Child over the age of 2 years
Angulation of 30 degrees at the fracture site
Weight over 12 Kg

References

Overhead skin traction with hip flexed 90 degrees and knee straight was described by Bryant
in 1873. When the child was over 2 years old or weighed more than 12 kg it often resulted in
vascular insufficiency. Skin traction works by friction between tape and skin. Damaged skin is
therefore a contraindication. Open fractures in children below 2 years are extremely
uncommon. Wound management is difficult with traction.

Peltier LF, A brief history of traction. JBJS Am 1968;50:1603-1617. Holmes SJK et al.
Domiciliary Gallows Traction for Femoral Shaft Fractures in Young Children. Feasibility,
Safety and Advantages. JBJS Br 1983; 65: 288-290. Rockwood and Wilkins Fractures in
Children, Beaty FH & Kasser JR (eds). Lippincott Williams and Wilkins 2005.
Question 9
Duchenne Muscular Dystrophy is associated with a mutation in the gene coding which
of the following proteins?
Tropomyosin
Actin
Myosin
Dystrophin
Calcitonin

References

Orthopaedic Surgery Essentials- Pediatrics Cramer & Scherl Publshed by Lippincott Wiliams
& Wilkins 2004
The Dystrophy of Duchenne. Jay V, Vajsar J. The Lancet Volume 357, Issue 9255, Pages
550 - 552, 17 February 2001
Duchenne Muscular Dystrophy (DMD) is the most common and most severe muscular
dystrophy of childhood. It is an X-linked recessive inherited disorder and is progressive
resulting in respiratory or cardiac failure and death in the early twenties. At present there is no
cure. The diagnosis is made by absence of the protein dystrophin on Immunohistochemical
analysis by immunoperoxidase staining of a muscle biopsy. The dystrophin-associated
protein complex consists of a chain of proteins that traverse the muscle cell membrane,
forming a link between the extracellular matrix and the intracellular actin. Defects in the
components of the dystrophin-glycoprotein complex have been noted in various muscular
dystrophies, including milder proximal myopathies, as well as disorders mimicking DMD in
their clinical severity. The detection of deletions or duplications by genetic studies by DNA
probes permit accurate carrier detection, family counselling, and prenatal diagnosis of DMD.

Question 10
Which of the following tendons most commonly ruptures in patients with rheumatoid
arthritis?
Flexor Pollicis Longus
Flexor Digitorum Superficialis
Extensor Digiti Minimi
Extensor Pollicis Longus
Extensor Digitorum Communis

References

Williamson L et al. Screening for Extensor Tendon Rupture in Rheumatoid Arthritis.


Rheumatology (oxford). 2001; 40: 420-3.

Question 11
When performing facet joint injection for the diagnosis of facet joint pain, which of the
following statements is correct?
Successful injection is usually associated with long-term pain relief
Each facet joint sends its nociceptive input through the medial branches of the dorsal
ramus
Pain relief on lumbar spine flexion following facet joint injection, is diagnostic for facet joint
pain
Is performed by injecting saline into the facet joint to reproduce pain
The procedure has a high risk of complications

References

Wilde V E, Ford J J, McMeeken J M. Indicators of lumbar zygapophyseal joint pain: survey of


an expert panel with the Delphi technique. Phys Ther 2007; (87): 1348-1361.
Orthopaedics Update, Spine, 2006
Campbells Operative Orthopaedics 10th Edition
Facet joint arthropathy is a recognised cause of low back pain. Facet joint injections were
common in the 70s and 80s but have been less popular recently as no good evidence exists
for their long term efficacy. They can be performed by either injecting local anaesthetic +/-
steroid directly into the joint or around the medial branches of the dorsal rami from the 2
adjacent levels as they emerge over the transverse processes. For example, the L4-5 facet
joint is anesthetized by blocking the L3 medial branch at the transverse process of L4, and
the L4 medial branch at the transverse process of L5.. Delphi survey published in Phys Ther.
2007 Oct, found pain eased by flexion, and exacerbated by extension to be one of the
sensitive for facet joint pain.) It s a safe procedure, with (no case reports of any significant
complications.

Question 12
Which of the following surgical techniques for spinal deformity carries highest risk of
tetraplegia/paraplegia?
Anterior thoracic surgery
Correction of rigid deformity by osteotomy
Posterior cervical surgery
Posterior spinal instrumentation
Anterior cervical surgery

References

Sucato DJ. Spinal Scoliotic Deformities. In: Vacarro A (ed), Core Knowledge in Orthopaedics:
Spine. Elsevier Mosby, Philadelphia, 2005.

Question 13
Which one of the following stems is true. If the thickness of a plate is doubled the
stiffness will increase by how much?
Twofold
Fourfold
Sixfold
Eightfold
Sixteenfold

References
Miller, fourth edition
Basic Orthopaedic Sciences; The Stanmore Guide Ed: Manoj Ramachandran
This is because Bending rigidity is equal to The Second moment area (SMA) of a structure
multiplied by the Young’s Modulus.
(Rigidity=SMA x Young’s Modulus)
The ‘Second Moment area’ is a variable which describes the spatial distribution of material
within a structure. It is independent of the type of material. A structure with a rectangular
cross section with width (w) and height (h) has a second moment area of wh3/12.
(SMA=wh3/12)
Therefore changing the material of a plate to one with twice the Young’s modulus (eg. cobalt
chrome rather than titanium) will double the rigidity while doubling the thickness of the plate
will increase the rigidity a factor of 8 (23).

Question 14
Linear elastic theory is used as a model for real material behavior. All of the following
are fundamental assumptions of this theory except
Stress and strain are not proportional to each other
Strain is reversible when the stress is removed
The material is insensitive to the rate of load application
Strain is internal deformation of the material.
The proportionality constant between stress and strain is called the modulus of elasticity.

References

Mow VC, Flatow EL, Ateshian GA. Biomechanics. Orthopaedic Basic Science. American
Academy of Orthopaedic Surgeons; 2002:148-158.
Basic Orthopaedic Sciences; The Stanmore Guide Ed: Manoj Ramachandran
The stress/ strain graph represents the behaviour of a material under a load. It consists of an
elastic region, where there is a linear (proportional) relationship between stress and strain
known as Hooke’s law, and a plastic region which occurs once deformation of the material is
irreversible. The point at which a material changes from elastic to plastic deformation is
known as the yield point. The plastic region of the curve does not obey Hooke’s law. The
Young’s modulus (modulus of elasticity) of a material applies to the gradient of the Elastic part
of the Stress/ Strain curve.

Question 15
For releasing acute compartment syndrome of intrinsic musculature of hand
Make 2 dorsal parallel incisions overlying the 2nd and 4th metacarpals
Make 4 dorsal parallel incisions overlying the 2nd, 3rd, 4th and 5th metacarpals
Make 4 dorsal incisions between the metacarpals
Make 3 dorsal parallel incisions between 2nd and 3rd metacarpals, 3rd and 4th metacarpals
and 4th and 5th metacarpals
Make as many incisions as required

References

Campbells Operative Orthopaedics


Compartment Syndromes of the Hand. Ouellette EA, Kelly R. JBJSAm Oct 1996, Volume 78-
A, Number 10
Compartment syndrome of the hand can be caused by crush injuries, multiple fractures,
burns, snakebites, but most commonly by iatrogenic injury from arterial lines or extravasated
IV medication.
There are 10 osteofascial compartments in the hand; 4 dorsal interossei, 3 palmar interossei,
thenar and hypothenar compartments(2), carpal tunnel (1). The dorsal and palmar interossei
are decompressed through 2 dorsal incisions overlying the 2nd and 4th metacarpals. The
fingers also have fascial compartments bounded by Cleland’s and Grayson’s ligaments which
are unyielding.
The intrinsic compartments can be decompressed via 2 dorsal incisions overlying the 2nd and
4th metacarpals with longitudinal slits in the fasciae dividing the dorsal and palmar interossei.
2 further incisions can be made overlying the thenar and hypothenar compartments to
decompress these compartments. Finally a carpal tunnel decompression can be performed.

Question 16
Which layer provides skin with most of its tensile strength?
Stratum corneum
Stratum granulosum
Stratum basale
Dermis
Hypodermis

References

Clinical Surgery in General (RCS Course Manual), Third Edition, Churchhill Livingstone
The skin is made up of two distinct layers; the epidermis and the dermis. Below the dermis is
the subcutaneous layer also known as the hypodermis. The epidermis in turn is divided into 5
distinct strata. From Superficial to deep they are:
Stratum Corneum
Stratum licidum
Stratum Granulosum
Stratum Spinosum
Stratum Basale
The Dermis is up to 40 times thicker than the epidermis and consists of two layers; the thin
papillary dermis and the reticular dermis which is where the majority of collagen fibres are
located making this layer the layer with the most tensile strength
With respect to lateral condyle fractures of the distal humerus in children which of the
following statements is true
The most common mechanism of injury is valgus stress on the extended elbow with the
forearm supinated
The Milch type 1 injury is a Salter Harris type 2 injury
The most commonly affected age group is 10-12 years.
The Milch type 1 injury is most common type of injury
Reduction manoeuvres for displaced fractures include flexion and supination

References

Type I: Fracture through the ossification center of the capitellum, lateral to the capitello-
trochlear groove, Salter-Harris type 4 injury. The elbow is usually stable and the relationship
between the forearm and the humerus remains intact
Type 2: Is the most common type of injury. Fractures through the capitello-trochlear groove,
Salter-Harris type 2 injury. Unstable. Peak incidence is at age 6 years. Management :
Undisplaced: controversial. Some authors advocate operative fixation, others suggest cast
immobilisation and weekly Xrays with reduction and fixation if there is subsequent loss of
position. Displacement - MUA (flex and pronate) + percutaneous wires or ORIF lateral
approach as appropriate.
Brinker Review of Orthopaedic Trauma. Saunders. Page 425

Question 18
All the statements regarding central cord syndrome are true EXCEPT
It most commonly results from an extension type mechanism
It most commonly affects patients over 50
It is the most common incomplete spinal cord injury syndrome
The cord is typically compressed between osteophytes anteriorly and the ligamentum flavum
posteriorly
There is greater loss of motor power and sensation to the lower extremities

References

Central cord syndrome (CCS), an acute cervical spinal cord injury (SCI), was initially
described by Schneider and colleagues in 1954. It is marked by a disproportionately greater
impairment of motor function in the upper extremities than in the lower ones, as well as by
bladder dysfunction and a variable amount of sensory loss below the level of injury.
Commonly over the age of 50, hyperextension injuries. Affects upper limbs more than lower
limbs motor and sensory loss. Prognosis - Fair.

Malone J et al. Physical Examination of the Spine. In: Vacarro A (ed), Core Knowledge in
Orthopaedics: Spine. Elsevier Mosby, Philadelphia, 2005.

Thompson JC. Netter’s Concise Atlas of Orthopaedic Anatomy. Elsevier, Philadelphia, 2002.

Question 19
The indication for manipulation with or without stabilization of a 5th metacarpal neck
fracture is
Apex dorsal angulation of more than 35 degrees.
Dominant hand injury in a 35 year old professional piano player.
Rotational deformity of the little finger.
Open fracture.
All of the above

References

Fractures of the metacarpals and phalanges constitute approximately 10% of all fractures.
Metacarpal fractures account for 30-40% of all hand fractures. Fractures of the 5th
metacarpal neck alone account for 10% of all fractures in the hand. Lifetime incidence of
metacarpal fractures is approximately 2.5%. Problems associated with metacarpal shaft
fractures relate to shortening, rotation, and dorsal apex angulation. Of these, malrotation is
the most critical. Minor rotational deformities can cause the fingers to overlap when the hand
is made into a fist. Rotational abnormalities are best judged clinically by comparing the injured
and uninjured digits through a full range of motion (ROM). With flexion, each digit should point
toward the scaphoid tuberosity. The plane of the nail should be similar between the injured
digit and the contralateral corresponding finger when evaluated in an intrinsic plus position.
Fracture splints should be forearm-based and should allow for motion of the interphalangeal
(IP) joints. Splints should extend over the dorsal and palmar aspect of the entire metacarpal
being treated. Generally, the wrist should be placed in 20-30° of extension; the MCP joints
should be immobilized in 70-90° of flexion, with the dorsal aspect of the splint extending to the
IP joints; and the volar aspect should end at the distal palmar crease. Buddy taping the
fingers of the involved metacarpal can aid in maintaining rotational control. After a short
period of immobilization, patients may be encouraged to use the fingers on the affected hand
to maintain motion. Most metacarpal neck fractures can be managed nonoperatively.
Requirements for operative fixation include severe angulation not treatable by closed means,
unstable rotational deformity, or significant comminution or bone loss, open fracture.
Operative treatment usually is best accomplished with closed reduction and percutaneous
pinning. Longitudinal pinning techniques or crossed pins are usually adequate to maintain
reduction. For very unstable fractures, internal fixation can be accomplished with dorsal
tension band wiring.

Question 20
Which of the following surgical techniques for treating Dupuytren's contracture has the
lowest rate of recurrence
Fasciectomy
Dermofasciectomy
Fasciotomy
Regional fasciectomy
Collagenase injection

References

Lubahn JD. Dupuytren’s Disease. In: Trumble TE et al (eds). Core Knowledge in Orthopaedic
Surgery: Hand, Elbow & Shoulder. Elsevier, Philadelphia, 2006.

Question 21
Which of the following is the most common complication following an interbody cage
fusion at the level of lumbar spine performed through an anterior approach?
Infection
Ileus
Implant migration
Haematoma and seroma
Atelectasis and pneumonia

References

McAfee PC. Interbody fusion cages in reconstructive operations on the spine, JBJS Am 1999;
81-A: 859-880.

Question 22
In wound healing, inflammation is triggered by
Neutrophils
Platelets
Monocytes
RBC
Lymphocytes
References

General and Systemic Pathology. Underwood JCE. Churchill Livingstone


The four stages of primary wound healing are haemostasis, inflammation, granulation and
remodelling.
Severing of blood vessels leads to arteriolar contraction, platelet aggregation and activation of
intrinsic and extrinsic clotting cascades resulting in the formation of thrombus. The platelets
release two potent growth factors, platelet derived growth factor (PDGF) and transforming
growth factor beta (TGFß) which are powerfully chemotactic for inflammatory cells including
macrophages and myofibroblasts.

Question 23
Which of the following does not occur when UHMWPE is sterilised using gamma
irradiation in a vacuum ?
Improved wear characteristics
Cross linking within the amorphous structure
Increased yield strength
Reduced toughness
Reduced Young's modulus

References

Gilbert JL, Cumber J, Butterfield A., “Surface micromechanics of ultrahigh molecular weight
polyethylene: Microindentation testing, crosslinking, and material behavior.” J Biomed Mater
Res. 2002 Aug;61(2):270-81.
Baker DA, Bellare A, Pruitt L., “The effects of degree of crosslinking on the fatigue crack
initiation and propagation resistance of orthopedic-grade polyethylene.” J Biomed Mater Res.
2003 Jul 1;66A(1):146-54.
orthoteers
Gamma sterilisation in vacuum causes crosslinking, which improves the oxidation and wear
resistance of polyethylene. However, it also alters the mechanical properties of UHMWPE.

Question 24
Which tendon is best “in phase” with the tibialis posterior tendon?
Flexor digitorum longus
Flexor hallucis longus
Tibialis anterior
Tendoachilles
Peroneus longus

References

OKU: Foot and ankle Ch 20, pg 214


Both FHL and FDL are synergistic to Tibialis posterior tendon but FDL is best “in phase” and
therefore is the preferred choice, whilst considering reconstructive options for Tibialis
posterior tendon insufficiency.

Question 25
When considering spinal infection, which of the following is false?
In children, inoculation of the intervertebral disc is haematogenous through persisting
vascular channels
In adults, the disc is avascular and so organisms invade from the metaphyseal region
adjacent to the disc
Infection may extend from the vertebral body to the epidural space and contiguous vertebral
bodies
There is no difference in the pathophysiology of spinal infection between children and
adults
Mycobacterium tuberculosis, although uncommon, is seen in immunocompromised
patients

References

Govender S, “Spinal infections”. JBJS [Br] 2005; 87(11):1454-8.


Stems C and E are true statements so this question tests if the candidate knows the
difference between adult and paediatric pathophysiology. This is different therefore D is the
false answer.

Question 26
In developmental dysplasia of the hip, a dislocated hip is suggested by the proximal
femoral ossific nucleus lying:
Below Shenton's line
Above Hilgenreiner's line
Medial to Perkins line
Above Klein's line
Medial to Nelaton's line

References

Significant DDH = 2 per 1000 live births Unstable hips at birth = 5-20 per 1000 F:M = 7:1 Left
more common than right. Risk factors: Female, Breech, First born, Oligohydramnios, Family
History . Associated with other 'packaging disorders' like torticollis (20%) & metatarsus
adductus (10%). In a child less than 6 months of age - It may be difficult to determine
between dysplasia and dislocation before walking age, there is no false acetabulum.
Shenton's line should be broken. Hilgenreiner's & Perkin's should be drawn and the hip
should be in the inner & lower quadrant between these lines if normal. The Acetabular Index
should be below 30 degrees at the age of one year. Metaphyseal Edge (ME) angle should be
negative, ie. the metaphyseal edge of the proximal femur should not be lateral to Perkin's line.
Centre Edge (CE) angle is useful in children older than 8 years of age; it is the angle between
the lateral border of the acetabulum & a vertical line through the centre of the femoral head.
Normal CE angle is more than 25 degrees . In an arthrogram: Limbus has a 'Rose thorn sign'
of inverted labrum between the femoral head & acetabulum. Hour glass constriction of
capsule - by psoas tendon. Capsular distension. Medial pooling of dye. Confirmation of
reduction after surgery by dye pooling less than 7mm & complete reduction with EUA &
arthrogram - no need for open reduction.
Orthoteers website. Lovell and Winter's Paediatric Orthopaedics
Dormans JP. Core Knowledge in Orthopaedics: Pediatric Orthopaedics. Elsevier,
Philadelphia. 2005.

Question 27
Which of the following is true about thoracic disc disease?
Symptomatic disc herniations are common in the thoracic spine.
An absolute indication for surgery is generally regarded to be the presence of severe and/or
progressive myelopathy.
Laminectomy is the standard treatment for disc prolapse
Majority of disc herniations in the thoracic region occur at upper thoracic levels (T4-T8).
Video-assisted thoracic surgery has fallen out of favour because of the higher complication
rates.

References

Introduction to Thoracic Disc Disease 1-Neurosurgical Focus, October 2000 Volume 9, Issue
4 TOPIC EDITOR: CHARLES B. STILLERMAN, M.D. 2-Neurosurg Focus 9(4), 2000. © 2000
American Association of Neurological Surgeons

Asymptomatic thoracic disc herniations are quite common in the general population with upto
37% reported in some MRI based studies. Symptomatic herniation, however, ranges from 1 in
1000 to 1 in 1 million persons. These are generally located at a lower thoracic level. The
general agreement is that surgery is indicated when myelopathic signs are present. These
patients may benefit from early surgery because the rate of recovery diminishes when more
advanced neurologic deficits are present. The earliest surgical approach, used in the early
1900s, was a posterior laminectomy. That technique was used for many years until numerous
studies demonstrated it produces poor results and has an unacceptable complication rate.
Video-assisted thoracoscopic approach has been reported to be a safe and efficacious
method of excising herniated thoracic discs. Follow-up results at 1 year resulted in high
patient satisfaction. VATS advantages include decreased length of hospitalization as well as
improved patient comfort.

Question 28
Following isolated posterolateral corner injury in which of the following positions is
the knee most unstable?
Extension
30 degrees flexion
60 degrees flexion
Arc from 30 to 90 degrees flexion
90 degrees flexion

References

Review of Orthopaedics, Miller. 4th Edition - p655


Isolated PLC injuries result in increasing instability most noted at 30o of flexion with instability
decreasing as the knee is flexed to 90o. Isolated PCL injuries cause greatest instability at 90o
of flexion. Combined PCL and PLC injuries result in increasing instability as the knee is flexed
from 30-90o.

Question 29
What is the commonest pathogen in superficial surgical site infection following total
hip replacement?
Pseudomonas
Coagulase negative staphylococci
Staphylococcus aureus
MRSA
Klebsiella

References

Health Protection Agency 2007, “Third report of the mandatory surveillance of surgical site
infection in orthopaedic surgery”
available from: http://www.hpa.org.uk/infections/topics_az/surgical_site_infection/default.htm
Accessed on 2/12/2007.
45% of surgical site infections are caused by Staphylococcus aureus (both methicillin
sensitive and methicillin resistant strains). Of these 62% were reported to be MRSA following
culture.

Question 30
Which one of the following statements about metabolic pathways is true ?
Calcium and phosphorus are always released together from bone. This release is stimulated
by parathyroid hormone and 1,25 dihydroxy Vitamin D3.
In acute and chronic metabolic acidosis urinary excretion of potassium ions is increased.
There is a wide fluctuation in the plasma concentration of calcium ions on a day to day
basis.
Urinary excretion of phosphate is not involved in the regulation of phosphate balance.
Calbindin, a calcium binding protein, is essential for intestinal calcium absorption.

References

Miller. Serum calcium ions are maintained within a tight normal range, total calcium may vary
with the albumin level. Calcium and phosphate metabolism is closely related. When the body
calls on the bone reservoir of calcium to increase the serum calcium, phosphate is also
released. This is stimulated by PTH and Vitamin D, A serum increase in phosphate is
prevented by increased urinary excretion following the action of PTH on the kidneys.
Alkalosis can cause hypokalaemia, not acidosis.
Calbindin is involved in active absorption of calcium from the duodenum, this is regulated by
vitamin D3. Passive absorption occurs in the jejunum and is not dependent on vitamin D3 or
calbindin.

Question 31
A 20 year old male presents with a first episode of anterior shoulder dislocation. It is
successfully reduced closed under sedation. Which is the injury most likely to be
associated with recurrence?
Rupture of the superior glenohumeral ligament
Impaction fracture of the antero-medial aspect of the humeral head
Impaction fracture of the postero-lateral aspect of the humeral head
Fracture of the greater tuberosity of the proximal humerus
A SLAP lesion

References

Ref 1. A comparison of the spectrum of intra-articular lesions in acute and chronic anterior
shoulder instability. Yiannakopoulos CK, Mataragas E, Antonogiannakis E. Arthroscopy. 2007
Sep;23(9):985-90.
Ref 2. Hovelius L, Augustini BG, Fredin H, Johansson O, Norlin R, Thorling J. Primary
anterior dislocation of the shoulder in young patients. A ten-year prospective study. J Bone
Joint Surg Am. 1996 Nov;78(11):1677-84.
Question Comments
Ref 1. In acute and chronic instability, the presence of a chondral or osteochondral Hill-Sachs
lesion was noted in 112 patients (88.1%), a Bankart lesion was noted in 106 patients
(83.46%), an anterior labroligamentous periosteal sleeve avulsion (ALPSA) lesion was noted
in 13 patients (10.23%), a SLAP lesion was noted in 26 patients (20.47%), a humeral avulsion
of the glenohumeral ligament (HAGL) lesion was noted in 2 acutely dislocated shoulders
(1.57%), and capsular laxity was noted in 33 patients (25.98%).
Ref 2. Radiographs, made for 185 shoulders at the time of the primary dislocation,
demonstrated an evident Hill-Sachs lesion in ninety-nine shoulders (54 per cent); this finding
was associated with a significantly worse prognosis with regard to recurrence than was no
evident lesion (p < 0.04).

Question 32
According to the PEP trial – Prevention of pulmonary embolism and deep vein
thrombosis with low dose aspirin, which of these statements about aspirin is true?
Significantly reduces the incidence of DVT in knee arthroplasty
Significantly reduces the incidence of pulmonary embolism in hip arthroplasty
Has a low complication rate in fracture and arthroplasty surgery
Produces a significant reduction in fatal pulmonary embolism and symptomatic DVT in
fracture neck of femur surgery
Produces a significant reduction in symptomatic DVT in hip arthroplasty surgery compared to
subcutaneous heparin.

References

PEP Trial Lancet. 2000 Apr 15;355(9212):1295-302. Previous trials of antiplatelet therapy for
the prevention of venous thromboembolism have individually been inconclusive, but a meta-
analysis of their results indicated reductions in the risks of deep-vein thrombosis and of
pulmonary embolism in various high-risk groups. The aim of this large randomised placebo-
controlled trial was to confirm or refute these apparent benefits. METHODS: During 1992-
1998, 148 hospitals in Australia, New Zealand, South Africa, Sweden and the UK randomised
13,356 patients undergoing surgery for hip fracture, and 22 hospitals in New Zealand
randomised a further 4088 patients undergoing elective arthroplasty. Study treatment was
160 mg daily aspirin or placebo, started preoperatively and continued for 35 days. Patients
received any other thromboprophylaxis thought necessary. Follow-up was of mortality and of
in-hospital morbidity up to day 35. FINDINGS: Among the patients with hip fracture, allocation
to aspirin produced proportional reductions in pulmonary embolism of 43% (95% CI 18-60;
p=0.002) and in symptomatic deep-vein thrombosis of 29% (3-48; p=0.03). Pulmonary
embolism or deep-vein thrombosis was confirmed in 105 (1.6%) of 6679 patients assigned
aspirin compared with 165 (2.5%) of 6677 assigned placebo, which represents an absolute
reduction of 9 (SE 2) per 1000 and a proportional reduction of 36% (19-50; p=0.0003). Similar
proportional effects were seen in all major subgroups, including patients receiving
subcutaneous heparin. Aspirin prevented 4 (1) fatal pulmonary emboli per 1000 patients (18
aspirin-group vs 43 placebo-group deaths), representing a proportional reduction of 58% (27-
76; p=0.002), with no apparent effect on deaths from any other vascular cause (hazard ratio
1.04 [95% CI 0.86-1.26]) or non-vascular cause (1.01 [0.84-1.23]). Deaths due to bleeding
were few (13 aspirin vs 15 placebo), but there was an excess of 6 (3) postoperative
transfused bleeding episodes per 1000 patients assigned aspirin (p=0.04). Among elective-
arthroplasty patients, rates of venous thromboembolism were lower, but the proportional
effects of aspirin were compatible with those among patients with hip fracture.
INTERPRETATION: These results, along with those of the previous meta-analysis, show that
aspirin reduces the risk of pulmonary embolism and deep-vein thrombosis by at least a third
throughout a period of increased risk. Hence, there is now good evidence for considering
aspirin routinely in a wide range of surgical and medical groups at high risk of venous
thromboembolism. PMID: 10776741 [PubMed - indexed for MEDLINE]

Question 33
In addition to the vascular sling formed by the sinus tarsi artery and the artery of the
tarsal canal, which of the following contributes significantly to the blood supply of the
talar body and is usually the only remaining blood supply to the talar body after
displaced talar neck fracture?
Posterior tubercle artery
Dorsalis pedis artery
Deltoid artery
Anterior tibial artery
Recurrent plantar artery

References
Ref 1. Gelberman RH, Mortensen WW. The arterial anatomy of the talus. Foot Ankle. 1983
Sep-Oct;4(2):64-72.
Ref 2. Peterson L, Goldie IF. The arterial supply of the talus. A study on the relationship to
experimental talar fractures.Acta Orthop Scand. 1975 Dec;46(6):1026-34.
Ref 1. The major blood supply to the body was provided by the artery of the tarsal canal. The
deltoid and sinus tarsi vessels provided significant minor sources of vascularity.
Ref 2. Ascending branches from the main arteries, arteria canalis tarsi and arteria sinus tarsi,
were ruptured in all cases. The vessels in the fracture area were all sharply cut off. Without
fracture displacement the surrounding arteries were remarkably intact, whereas with
displacement these became affected by varying degrees of disruption. The vessels in the
bone adjacent to the fracture remained intact.

Question 34
Using the principles of the Lauge-Hansen classification, how would you reduce an
ankle with the following fracture pattern: a spiral fracture of the distal fibula through
the syndesmosis with a transverse medial malleolar fracture.
Pronation of the foot and internal rotation
Pronation of the foot and abduction
Supination of the foot and adduction
Supination of the foot and internal rotation
Pronation of the foot and external rotation

References

A. Paige Whittle, George W. Wood II. Fractures of Lower Extremity. Campbell's operative
orthopaedics.10th Edn. Editor S Terry Canale. Chapter 51.
To reduce fractures an opposite force should be applied to the one that caused the fracture.
According to the Lauge-Hansen classification the above fracture pattern is due to supination-
external rotation. Therefore pronation and internal rotation is the reduction manoeuvre for this
fracture.

Question 35
With regard to childhood spondylolisthesis
Spondylolisthesis in children is commonly associated with quadriceps tightness
90% of children with spondylolisthesis will experience signficant back pain
It is associated with spina bifida occulta.
It is usually of the dysplastic type
Severe (>50%) slips should be reduced before fusion

References

Spondylolisthesis in children is associated with hamstring tightness. It is often painless. It is


most commonly Isthmic (type II). Reduction of spondylolisthesis is controversial as it has
been associated with a 20-30% incidence of L5 nerve root injury. Review of Orthopaedics.
Ed. Miller. 4th Ed. p427-30.

Question 36
Where is the hypovascular area of the peroneus brevis tendon located?
At the musculotendinous junction
Proximal to fibular groove
At the level of the fibular groove
Distal to the fibular grove
At its insertion on the 5th metatarsal base

References

Petersen W et al: Blood supply of the peroneal tendons: Injection and immunohistochemical
studies of cadaver tendons. Acta orthop Scand 2000; 71:168-174
Question Comments
In the region where the peroneus brevis tendon passes through the fibular groove, the
longitudinally-oriented intratendinous vascular network is interrupted and the tendon is almost
avascular. In this region, the tendon is squeezed between the peroneus longus tendon and
the bony slide bearing of the lateral malleolus.

Question 37
Hypercalcaemia may present with all of the following clinical signs EXCEPT:
Abdominal cramps
Trousseau's sign
Polyuria
Pyschosis
Nephrolithiasis

References

Trousseau's sign of carpal spasm is present in hypocalcaemia and is due to the


hyperexcitability of muscles. After inflation of sphygmomanometer to 20 mm of Hg over
systolic blood pressure, ischaemia of the ulnar and median nerves occurs. Similarly,
Chovostek's sign of facial spasm is elicited by tapping over the facial nerve approximately 20
mm anterior to the ear lobe below the zygomatic arch.

The symptoms of hypercalcaemia are of “bones, stones, groans and psychiatric moans”, due
to the symptoms of fatigue, depression, confusion, anorexia, nausea, vomiting, constipation,
pancreatitis or increased urination. If it is chronic it can result in urinary calculi (renal or
bladder stones). Abnormal heart rhythms can result, and ECG findings of a short QT interval
and a widened T wave suggest hypercalcaemia.

Wikipedia
A 25 year old male sustains a closed fracture of the shafts of the radius and ulna
following a fall from his motorcycle. Which of the following is true regarding surgical
management?
Good Interfragmentary compression of fractures can be achieved with locking plates
If compartment syndrome develops, forearm fasciotomies should include the carpal tunnel

Routine bone grafting is indicated in plate fixation of forearm fractures


Low contact - dynamic compression plating reduces plate contact with bone by 25%
In Galeazzi fractures, anatomical reduction of the radius rarely reduces the distal radio-ulnar
joint
References

Ref 1. Stevens CT, ten Duis HJ. Plate osteosynthesis of simple forearm fractures: LCP versus
DC plates. Acta Orthop Belg. 2008 Apr;74(2):180-3.
Ref 2. M. M. McQueen and C. M. Court-Brown Compartment Monitoring In Tibial Fractures:
The Pressure Threshold For Decompression. J Bone Joint Surg Br, Jan 1996; 78-B: 99 - 104.
Ref 3. Anderson LD, Sisk TD, Tooms RE et al. Compression-plate fixation in acute diaphyseal
fractures of the radius and ulna. J Bone Joint Surg1975; 57A:287–297.

Ref 4. Post-traumatic radioulnar synostosis after forearm fracture osteosynthesis. Bauer G,


Arand M, Mutschler W. Arch Orthop Trauma Surg. 1991;110(3):142-5.
Ref 1. In a study comparing the time to radiological bony union of simple A-type fractures of
the forearm, treated with either a locking compression plate (LCP) or a dynamic compression
plate (DCP), the compressed fractures, irrespective of the type of plate, healed 10 weeks
faster than the non-compressed fractures.

Ref 2. In a prospective series of 116 tibial diaphyseal fractures followed up for a mean of 15
months, the use of a differential pressure of 30 mmHg as a threshold for fasciotomy led to no
missed cases of acute compartment syndrome.
Ref. 3. Almost 100% union rate was achieved in a series of 244 patients with selective bone
grafting only when the comminution was more than on third of the circumference of the shaft.
This paper also references previous series where forearm fractures treated without internal
fixation resulted in upto 85% unsatisfactory results.
Ref 4. Synostosis was noted in 5 of 12 patients who had their proximal radius and ulna
fractures fixed through a single Boyd approach.

Question 39
In mobile, mentally able, elderly patients with a displaced intracapsular fractured neck
of femur, the following statements are true EXCEPT
Reoperation rate for internal fixation is higher than for total hip replacement.
The direct lateral approach (Hardinge) has a lower risk of dislocation compared with the
posterior approach
Internal fixation is more cost effective in long term than total hip replacement.
Function and pain are significantly better in patients with arthroplasty than internal
fixation.
An RCT showed the mortality rate is similar in patients managed with hip replacement or with
internal fixation

References

Blomfeldt R et al Comparison of internal fixation with total hip replacement for displaced
femoral neck fractures. Randomized, controlled trial performed at four years. J Bone Joint
Surg Am. 2005 Aug;87(8):1680-8.
Both groups had a mortality of 25% at 4 years. The rate of hip complications and reoperation
was significantly lower in the THR group. Hip function and quality of life was significantly
better in the THR group.
Enocson et al. Dislocation of hemiarthroplasty after femoral neck fracture: better outcome
after the anterolateral approach in a prospective cohort study on 739 consecutive hips. Acta
Orthop 2008
Anterolateral approach lowest rate of dislocation. Odds ratio for dislocation with posterior
approach and repair 3.9, posterior approach with no repair 6.9

Johansson T et al The total costs of a displaced femoral neck fracture: comparison of internal
fixation and total hip replacement. A randomised study of 146 hips. Int Orthop. 2006
Feb;30(1):1-6. Epub 2005 Dec 23.
143 patients over 75, 34 of 78 who were internally fixed went on to secondary surgery. 12 of
68 THRs dislocated, the majority of whom were mentally impaired. Harris hip score was
higher and pain lower in THR group. Over two years, the total hospital costs were equivalent
for THR and internal fixation.

Question 40
The following is NOT a characteristic of viscoelastic materials:
Stiffness decreases as the strain rate increases
Loses energy during each cycle of loading
Deforms over time with a constant load
Stress decreases with a constantly applied strain
None of the above.

References

A viscoelastic material is sensitive to the speed at which the load is applied. In general the
faster the strain rate (rate of loading) the higher the stiffness (the stress at a given level of
strain). Viscoelastic materials have the following properties: Creep - deformation of a material
over time to a constant load. Stress Relaxation - with a constantly applied strain the stress in
the material decreases. Hysteresis - under cyclical loading there is loss of energy in the
material during each cycle. Basic Orthopaedic Sciences, Manoj Ramchandran Orthoteers
website

Question 41
Which of the following factors correlates MOST with poor outcome from ulnar nerve
decompression surgery for entrapment neuropathy in cubital tunnel?
Occupation
Diabetes
Age of patient
Results of EMG studies
Intrinsic muscle atrophy

References

Cubital Tunnel Syndrome occurs with ulnar nerve entrapment as it crosses the elbow. Careful
examination is required to differentiate this condition from nerve entrapment at the wrist -
ulnar tunnel syndrome.

The ulnar nerve arises from the medial cord of the brachial plexus containing fibers from the
C8 and T1 nerve roots. At its proximal aspect, the ulnar nerve lies medial to the
axillary/brachial artery to the middle third of the humeral shaft. At this point, it passes through
the intermuscular septum (arcade of Struthers) and along the medial head of the triceps
muscle towards the ulnar groove (posterior to the medial epicondyle).

Within the cubital tunnel, the ulnar nerve lies beneath the fascial arcade between the two
heads of the flexor carpi ulnaris (FCU) and anterior to the flexor digitorum profundus (FDP).
Proximally, the ulnar nerve sends off an articular branch before innervating the FCU and the
ulnar head of the FDP muscle.

Compression of the ulnar nerve commonly occurs between the ulnar and humeral heads of
the FCU just distal to the medial epicondyle (Osborne's fascia). Other sources of compression
include the arcade of Struthers, medial head of the triceps, the aponeurosis of the FCU,
synovial hypertrophy, tumor (ganglia or lipomata), aberrant muscles (anconeus
epitrochlearis), cubitus valgus, bone spurs and with nerve subluxation during elbow flexion.

Patients often complain of pain at the medial aspect of the elbow. There is loss of sensation
to light touch and vibratory sense at the small and ulnar half of the ring finger, weakness of
grip strength and in severe cases, wasting of the intrinsic musculature. With the elbow in
flexion, Tinel's sign is positive at the elbow with exacerbation of symptoms during
compression (or percussion) near the origin of the FCU (often within one minute).

Ulnar neuropathy consists of minor hypesthesia and paresthesia


Hypesthesia with weakness of the interossei
Complete/partial loss of sensitivity with marked weakness and wasting of the interossei

Electrodiagnostic studies are useful in determining the location of nerve compression


Significant compression shows a conduction delay greater than 33%

Activity modification, splinting in extension and use of nonsteroidal anti-inflammatory drugs


may aid in alleviating symptoms; 50% improve spontaneously. Steroid injections are rarely
used because of risk of direct injury to the nerve.

Surgical treatment for refractory cases includes:


Simple decompression
Subcutaneous transposition
Intramuscular transposition
Submuscular transposition or medial epicondylectomy
Decompression, in general, is appropriate for Grade I and II neuropathy, while anterior
transposition is generally performed for patients with Grade III involvement. In addition to
those with severe symptoms, anterior transposition is preferred for cases involving bony
deformity or nerve subluxation.

In chronic palsy (>3-4 mo in duration) associated with pain, muscle weakness, and/or
atrophy, surgical outcome is less certain. Duration of entrapment and severity of numbness
and muscle weakness are important factors in prognosis. Improvement may be limited or may
not occur following decompression and transposition in these chronic cases, but further
progression can be halted with proper decompression.

Miller fourth edition

Question 42
Which of the following statements is TRUE regarding the treatment of displaced
intracapsular fractures of the hip in healthy older patients?
Secondary surgery is most common after bipolar hemiarthroplasty
Long-term results of total hip replacement may be better than those of bipolar
hemiarthroplasty
Functional outcome at 24 month after surgery is best after bipolar hemiarthroplasty
Fracture reduction and fixation is more cost effective than bipolar hemiarthroplasty
Bipolar hemiarthroplasty gives a similar quality of life score to fracture reduction and fixation
at 12 months

References

Randomized Comparison of Reduction and Fixation, Bipolar Hemiarthroplasty, and Total Hip
Arthroplasty. Treatment of Displaced Intracapsular Hip Fractures in Healthy Older Patients
J.F. Keating, A. Grant, M. Masson, N.W. Scott, J.F. Forbes The Journal of Bone and Joint
Surgery (American). 2006;88:249-260.
Multicentre RCT, mortality equivalent in the three groups. The secondary surgery rate was
39% in the fixation group, 5% in the bipolar group and 9% in the THR group. THR had the
best functional outcome at 24 months, the fixation group had the worst hip questionnaire and
EuroQol scores at 4 and 12 months.

Question 43
Which of the following statements is FALSE regarding the pivot shift test?
The tibial condyle subluxes anteriorly in an extended position
The MCL should be intact to elicit the test
The ilio-tibial band should be intact to elicit the test
All the manoeuvres are similar to the reverse pivot shift test except that the knee is stressed
in varus
The knee is extended initially and then flexed to reduce the subluxed tibial lateral condyle.

References

The pivot shift test is to assess the integrity of the anterior cruciate ligament. The knee is
moved from extension into flexion, with the knee held in internal rotation whilst maintaining a
valgus strain.
The reversed pivot shift test is to assess the integrity of the posterior cruciate ligament. The
test is identical except the knee is held in external rotation..
The ACL is the primary restraint to anterior tibial displacement, and a secondary restraint to
tibial rotation. If the ACL is deficient, when the knee is extended with a valgus force and
internal rotated, the lateral tibial condyle subluxes anteriorly. This subluxation is reduced as
the knee is flexed to more than 30 degrees, when the iliotibial band becomes a knee flexor
instead of an extensor.
The test requires intact medial structures as well as an intact iliotibial band.

Question 44
Concerning flexor tendon repairs in the hand, which of the following statements is
FALSE?
The repair is weakest between days 6 and 12
Circumferential epitenon repair at the rupture site in addition to core suture improves strength
of the repair by 20%.
FDP bony avulsions can be successfully repaired weeks after injury
The repaired flexor pollicis longus tendon has a lower rerupture than long finger flexor
tendons
Maximum strength is achieved by 8 months.

References

The goal of an ideal repair is restoration of the tendon to allow function. Strickland noted the
characteristics of the ideal primary flexor tendon repair include: Sutures that are easily placed
in the tendon, secure suture knots, smooth juncture of tendon ends, minimal gapping at the
repair site, minimal interference with tendon vascularity and sufficient strength throughout
healing to permit the application of early motion stress to the tendon.
Gap formation is the initial event in flexor tendon repair failure. Gapping of less than 3 mm will
minimize adhesion formation and reduce the work of flexion. Gapping can be minimized with
suture techniques that provide for the requisite suture tendon interface grasp and materials
that reduce material stretch.
The strength of the repair has a linear relationship to the number of strands crossing the
repair, and repairs most often rupture at the knots. Although multi-strand repairs are stronger,
they have the disadvantage of being technically difficult, especially when resecting the critical
A2 and A4 pulleys. Tensile strength requirements for early active motion rehabilitation
protocols are a minimum of a four-strand, and preferably six-strand repair.
The number of suture strands is more significant than the number of grasping loops, loops
tend to collapse under load and result in gapping.
A circumferential epitendinous suture not only improves tendon gliding but also improves the
tensile strength of the repair (by 20%) and allows for less gap formation.
Some of the popular suture methods include the Kessler, the modified Kessler, Tajima
modification, Tsuge looped suture, Strickland, four-strand cruciate, Becker, and Savage.
Dorsal rather than a palmar placement of core suture results in a stronger repair6, but this
may compromise the vincular vascular supply that originates dorsally.
Large calibre sutures also increase the strength of the repair although they may increase the
width of the repair and increase the work of flexion. There is no definite answer as to the best
suture material.
The fewer the number of suture knots involved in the repair site the better because repairs
usually rupture at the knots. Additionally, knots should be located outside the repair because
they are generally stronger and less likely to interfere with tendon gliding. Knots should be
buried and never tied on the outer surface of the tendon as they can interfere with tendon
gliding.
Benefits of tendon sheath repair include reduced adhesion formation, return of synovial
nutrition, and better tendon mechanics. Repair can be technically demanding, however, and
there is also the risk of making the repaired sheath too narrow and, as a result, restricting
tendon movement.
Schuind et al performed in vivo experiments to measure digital flexor tendon tensile strength.
They described an average tensile load of 8.8 N in the flexor digitorum superficialis (FDS)
during passive motion, and forces averaging 17 N with active pinch of the index finger.
Regardless of the exact numbers, techniques in modern flexor tendon repair are providing
strong enough repairs to allow for protective passive and light active motion rehabilitation
protocols, and the results have been good. Postoperatively immobilization will prevent
rupture, however, can lead to the formation of adhesions. The repair is weakest between
postoperative days 6 and 12.
Clinical studies of zone II repairs have shown rupture rates as high as 46%. The flexor
pollicus longus tendon has a rerupture rate of 20% in some series, compared with 2-5% in
fingers.
Campbell’s orthopaedics, Miller 4th Edition

Question 45
Bleeding tendency is least likely in which one of the following conditions
Ehlers – Danlos syndrome
Osteopetrosis
Rheumatoid arthritis
McCune Albright syndrome
Protein S deficiency

References

The bleeding tendency in Ehlers Danlos syndrome is due to a defect in collagen III leading to
vessel fragility.
Hoffbrand AV & Pettit, JE. Essential Haematology, 3rd Edition, 1997 (Publishers: Blackwell
Scientific).
Bleeding can occur in osteopetrosis due to bone marrow failure.
Rheumatoid arthritis is commonly associated with anaemia of chronic disease. It can lead to
bleeding due to medication side effects, eg. NSAIDS or post splenectomy.
McCune Albright syndrome can cause bleeding due to platelet function abnormalities. It may
also presents with precocious puberty.and vaginal bleeding in girls as young as six months.
Protein S deficiency is a thrombophilia. Protein S is a vitamin K dependent anticoagulant
protein.

Question 46
Which of the following muscles/tendons is NOT involved in intersection syndrome?
Extensor pollicis brevis
Extensor pollicis longus
Extensor carpi radialis longus
Extensor carpi radialis brevis
Abductor pollicis longus

References

Grundberg AB, Reagan DS. Pathologic anatomy of the forearm: intersection syndrome. J
Hand Surg [Am]. 1985 Mar;10(2):299-302.

Question 47
Which of the following materials exhibit the steepest gradient on the linear section of
the stress-strain graph? (With stress on the vertical axis and strain on the horizontal
axis)
Cortical Bone
Polyethylene
PMMA (polymethylmethacrylate)
Titanium
Stainless steel

References

Review of Orthopaedics, Miller (4th Edition, p129


Stress = Force/Area
Strain = change in length/original length.
Young's Modulus of Elasticity = stress/strain(which is the slope in the elastic range of the
curve))

Question 48
Which investigation is considered 'gold standard' in diagnosing deep vein thrombosis
after total hip replacement?
Duplex Ultrasonography
125 I-labeled Fibrinogen
Venography
D-Dimer
Impedence plethysmography

References
Venography has upto 97% accuracy. D-Dimer and 125 I-labeled Fibrinogen from operative
site artefact causes false positive. Impedence plethysmography has poor sensitivity. Duplex
Ultrasonography although useful as initial tool, has only 90% accuracy. Miller. Review of
orthopaedics, third edition. Basic Science, page 106

Question 49
Increasing the diameter of an intramedullary nail by 10% will increase its bending
10%
25%
50%
75%
100%

References

Bending rigidity of an IM nail is related to the fourth power of the nail's radius. Increasing
radius by 10% will increase bending rigidity by 50%. Miller M Review of orthopaedics Fourth
Edition

Question 50
Which organism is most associated with human bite infections?
Eikenella corrodens
Pasturella multocida
Enterococcus spp
Pseudomonas aeruginosa
Acromonas hydrophilia

References

Tonta, Katherine ; Kimble, Frank W. Human bite injuries of the hand: The Tasmanian
experience. ANZ Journal of Surgery. 71(8):467-471, August 2001.
Green’s Operative Hand Surgery.5th Edition pp85-86.
Four mechanisms of human bite injury:
1. Self inflicted: nail biting/ sucking an open wound
2. Traumatic amputation secondary to a bite injury
3. Full thickness bite injury to other parts of hand
4. Fight bite: clenched fist into mouth

Question 51
Hypercalcaemia can be caused by all of the following conditions EXCEPT:
Hyperthyroidism
Multiple myeloma
Hypervitaminosis D
Bisphosphonate usage
Tertiary hyperparathyroidism
References

Hypercalcaemia Initially symptoms are nonspecific Excess urine and thirst Loss of appetite
Depression Muscle weakness Anorexia and nausea Constipation Fatigue At higher levels,
Abdominal pain Vomiting Dehydration Lethargy Heart problem Pancreatitis Osteoporosis
Coma Causes are Primary hyperparathyroidism (female 50-60) Malignancy is the most
common cause of hypercalcaemia Granulomatous conditions -such as sarcoidosis and
tuberculosis Endocrine conditions - such as thyrotoxicosis, phaeochromocytoma and primary
adrenal insufficiency Drugs- such as thiazide diuretics, vitamin D and vitamin A supplements
Familial- e.g. familial hypocalciuric hypercalcaemia Tertiary hyperparathyroidism - Post
Kidney transplant or chronic dialysis Other: e.g. prolonged immobilization, milk-alkali
syndrome, AIDS. Treatment for acute hypercalcaemia should be initiated in hospital and will
include increasing the circulating volume and hydration with saline. Once circulating volume is
normal, addition of a loop diuretic e.g. furosemide Biphosphonates such as pamidronate and
zoledronic acid or salmon calcitonin may be used to reduce bone turnover. Plicamycin,
gallium nitrate and prednisolone are also occasionally used in specialist settings. In severe
cases haemodialysis may be effective. Asymptomatic patients with PTH mediated
hypercalcaemia which doesn't meet the recognised criteria for surgery may be treated
conservatively with regular monitoring of bone density, renal function and serum and urinary
calcium levels. Reduce patients intake of calcium. Mobilize a bed bound patient

Question 52
Which of the following statements concerning the Weaver Dunn procedure is correct?
A loop of PDS tape is passed around the coracoid.
Primary repair of the coraco-clavicular ligament is performed
A screw is passed from the clavicle to the coracoid.
The distal clavicle is excised.
An intra-medullary wire is used to stabilise the acromio-clavicular joint

References

Treatment of acromio-clavicular injuries, especially complete acromio-clavicular separation.


Weaver JK, Dunn HK. J Bone Joint Surg Am. 1972;54;1187-1194

Question 53
Enchondromas in the hand
Occur most commonly in the distal phalynx
Usually affects the epiphysis
Histological appearances demonstrate a distinct cartlilaginous cap
Radiographic features include endosteal scalloping and calcifications within the lesion
Lesions usually appear in the skeletally immature

References

Enchondroma is the most common primary neoplasm that occur in the skeleton of the hand.
Enchondromas are most often discovered incidentally or when a pathological fracture occurs
through a solitary lesion in the diaphysis or metaphysis of a phalynx in a young adult. Lesions
usually appear in the skeletally mature in the second to forth decades of life
most commonly affects the proximal and middle phanlanges and the metacarpals.
Involvement of the epiphysis is rare. Growth usually starts in the metaphysis, extending into
the diaphysis.
Histology demonstrates benign cartilage. Cartilaginous cap is typical of osteochondroma.

Question 54
Swanson's classification of congenital hand deformities includes all of the following,
except
Genetic anomaly
Failure of differentiation or separation of parts
Congenital constriction
Failure of formation of parts
Undergrowth (hypoplasia)

References

Swanson AB J Hand Surg 1983 8A 693-702 Oxford Textbook of Orthopaedics and Trauma
p2500

Question 55
All of the following substances released in response to acute blood loss are powerful
endogenous vasoconstrictors EXCEPT
Vasopressin
Angiotensin
Adrenaline
Aldosterone
Noradrenaline

References

BERNE, R., LEVY, M. 2000. Principles of Physiology. Mosby, p.293.


Skøtt O et al.. Rapid actions of aldosterone in vascular health and disease—friend or foe?
Pharmacology & Therapeutics. Volume 111, Issue 2, August 2006, Pages 495-507
The catecholamines adrenaline and noradrenaline are released from the adrenal medulla.
Vasopressin is released from the posterior pituitary gland.
Angiotensin is produced by the action of renin on the plasma protein angiotensinogen in
response to acute blood loss.
All are powerful vasoconstrictors.
Aldosterone is released by the adrenal cortex in response to angiotensin and stimulates
active sodium and passive water reabsorption by the renal tubules.

Question 56
What is the earliest time electrodiagnostic studies can reliably interpreted after injury?
Within 24 Hours
24-48 Hours
3-5 Days
5-7 Days
10-14 Days
References

Green’s Operative Hand Surgery.5th Edition pp.1081-2.


Orthoteers website
http://www.emedicine.com/neuro/topic382.htm
A carefully planned and executed electrodiagnostic study is paramount in the evaluation of
nerve injuries. Needle EMG can demonstrate whether the injury is complete or incomplete at
any time after injury. Nerve conduction studies are required to differentiate demyelination from
axon loss; they yield the maximal information in this regard approximately 10 days after the
injury. Nerve conduction studies should be bilateral to allow side-to-side comparisons of
amplitude.

Question 57
Concerning haemophilia which one of the following statements is TRUE
Haemophilia B is due to a deficiency in factor VIII
Haemarthroses and muscle haematomas are common in heterozygous von Willebrand’s
disease
Haemophilia B is treated with infusions of factor IX
Spontaneous haemarthrosis secondary to haemophilia B can be controlled if factor VIII levels
are raised to 10% of normal
Operative and post traumatic haemorrhage is not life threatening in mildly affected
patients

References

Haemophilia A is factor VIII deficiency and B is factor IX deficiency. Hoffbrand AV & Pettit, JE.
Essential Haematology, 3rd Edition, 1997 (Publishers: Blackwell Scientific)

Question 58
Which of the following best characterises the chronic phase (>12 months) of reflex
sympathetic dystrophy (complex regional pain syndrome)as described by Lankford
and Evans
Pain, swelling, redness, normal radiographs
Diminished pain, glossy dry skin, fibrosis, osteopenia
Worse pain, dry skin, cyanosis, osteopenia
Decreased range of movement, hyperhydrosis
No pain, pink skin, return of normal range of movement

References

Review of Orthopaedics,Miller, 4th edition Page 381


Green’s Operative Hand Surgery.5th Edition pp.2015-2048.
RSD is part of the broader category of Complex regional pain syndrome.
CRPS was introduced as RSD lacks pathognomonic criteria.

Question 59
An 11 year old girl who enjoys gymnastics presents with acute lower back pain
resistant to simple analgesia. There is no lower limb neurological deficit. Her CRP is
<8. Which of the following is the most appropriate investigation to arrange next?
Whole spine MRI
CT scan
Plain anteroposterior and lateral radiograph lumbar spine
Bone scan
ESR

References

Peadiatrics. Orthopaedic Surgery Essentials. Cramer and Scherl. 2004. pp. 69-73
History very important: osteoid osteoma - , waking at night responding to NSAIDS; overuse
syndromes (gymnastic), - spodylolysis; neurological complaints
Physical exam first including gait, neurology, Adams forward bending test.
Then plain x-rays (with oblique if spodylolysis suspected)
If x-rays normal organise Bone Scan

Question 60
In a seventy year old man requiring an anterior cervical discectomy and fusion, the
common carotid artery divides into internal and external carotid arteries at what level ?
Upper border of the thyroid cartilage
The carotid tubercle
The hyoid cartilage
The cricoid cartilage
At the level of the stellate ganglion

References

The principal arteries of supply to the head and neck are the two common carotids; they
ascend in the neck and each divides into two branches, viz., the external carotid, supplying
the exterior of the head, the face, and the greater part of the neck; the internal carotid,
supplying to a great extent the parts within the cranial and orbital cavities.

The Common Carotid Artery (A. Carotis Communis)—The common carotid arteries differ in
length and in their mode of origin. The right begins at the bifurcation of the innominate artery
behind the sternoclavicular joint and is confined to the neck. The left springs from the highest
part of the arch of the aorta to the left of, and on a plane posterior to the innominate artery,
and therefore consists of a thoracic and a cervical portion.
The thoracic portion of the left common carotid artery ascends from the arch of the aorta
through the superior mediastinum to the level of the left sternoclavicular joint, where it is
continuous with the cervical portion.

The external carotid artery begins opposite the upper border of the thyroid cartilage, and,
taking a slightly curved course, passes upward and forward, and then inclines backward to
the space behind the neck of the mandible, where it divides into the superficial temporal and
internal maxillary arteries. It rapidly diminishes in size in its course up the neck, owing to the
number and large size of the branches given off from it. In the child, it is somewhat smaller
than the internal carotid; but in the adult, the two vessels are of nearly equal size. At its origin,
this artery is more superficial, and placed nearer the middle line than the internal carotid, and
is contained within the carotid triangle.
The internal carotid artery supplies the anterior part of the brain, the eye and its appendages,
and sends branches to the forehead and nose. Its size, in the adult, is equal to that of the
external carotid, though, in the child, it is larger than that vessel.

Question 61
What is the primary wear mechanism of polyethylene components in total knee
arthroplasty ?
Adhesive and abrasive
Fatigue and delamination
Crevice corrosion
Oxidative degradation
Volumetric

References

CURRENT CONCEPTS REVIEW: THOMAS P. SCHMALZRIED and JOHN J. CALLAGHAN.


Wear in Total Hip and Knee Replacements. J. Bone Joint Surg. Am., Jan 1999; 81: 115 - 136.
Differences in the articulating surfaces and motion patterns of total knee replacements as
compared with those of total hip replacements have important effects on the wear of the
polyethylene. Decreased conformity can result in substantially increased contact stresses that
can exceed the yield strength of polyethylene. Furthermore, in a total knee replacement, the
motion pattern can include rolling, sliding, and rotation on the same surface. The combination
of these factors results in differences in the mechanisms of wear for total knee replacement
compared with those for total hip replacement. In total hip replacement, the predominant wear
mechanisms appear to involve microadhesion and microabrasion with the generation of many
polyethylene particles less than one micrometer in length. In contrast, subsurface
delamination, pitting, and fatigue cracking, with the release of much larger particles of
polyethylene, have been recognized as important mechanisms of wear in total knee
replacement. These mechanisms result in the visually striking surface damage of some
retrieved tibial polyethylene bearings.

Question 62
Of the following which is the most important factor for stability of a fracture treated
with external fixation
Larger diameter pins
Fracture reduction and bone to bone contact
Increased spacing between pins
Decreased bone to rod distance
Pins in different planes
References

Miller M
Review of Orthopaedics
Fourth Edition

Question 63
A reduction in plasma sodium ion concentration will result in an increase in production
and release of
Atrial Natriuretic Hormone
Renin
Parathormone
Urine
Sweat

References

Serum sodium concentration is regulated by stimulation of thirst, secretion of ADH, feedback


mechanisms of the renin-angiotensin-aldosterone system, and variations in renal handling of
filtered sodium. Increases in serum osmolarity above the normal range (280-300 mOsm/kg)
stimulate hypothalamic osmoreceptors, which, in turn, cause an increase in thirst and in
circulating levels of ADH. ADH increases free water reabsorption from the urine, yielding urine
of low volume and relatively high osmolarity and, as a result, returning serum osmolarity to
normal. ADH is also secreted in response to hypovolemia, pain, fear, nausea, and hypoxia.

Aldosterone, synthesized by the adrenal cortex, is regulated primarily by serum potassium but
also is released in response to hypovolemia through the renin-angiotensin-aldosterone axis.
Aldosterone causes absorption of sodium at the distal renal tubule. Sodium retention
obligates free water retention, helping to correct the hypovolemic state. The healthy kidney
regulates sodium balance independently of ADH or aldosterone by varying the degree of
sodium absorption at the distal tubule. Hypovolemic states, such as hemorrhage or
dehydration, prompt increases in sodium absorption in the proximal tubule. Increases in
vascular volume suppress tubular sodium reabsorption, resulting in natriuresis and helping to
restore normal vascular volume. Generally, disorders of sodium balance can be traced to a
disturbance in thirst or water acquisition, ADH, aldosterone, or renal sodium transport.

Hyponatremia is physiologically significant when it indicates a state of extracellular


hyposmolarity and a tendency for free water to shift from the vascular space to the
intracellular space. Although cellular edema is well tolerated by most tissues, it is not well
tolerated within the rigid confines of the bony calvarium. Therefore, clinical manifestations of
hyponatremia are related primarily to cerebral edema. The rate of development of
hyponatremia plays a critical role in its pathophysiology and subsequent treatment. When
serum sodium concentration falls slowly, over a period of several days or weeks, the brain is
capable of compensating by extrusion of solutes and fluid to the extracellular space.
Compensatory extrusion of solutes reduces the flow of free water into the intracellular space,
and symptoms are much milder for a given degree of hyponatremia.

When serum sodium concentration falls rapidly, over a period of 24-48 hours, this
compensatory mechanism is overwhelmed and severe cerebral edema may ensue, resulting
in brainstem herniation and death.

McGeown JG. 2002 Physiology Churchill Livingstone


Question 64
The Akin osteotomy used in hallux valgus correction
Is commonly chosen for incongruent joints.
Is not a periarticular procedure.
Is routinely carried out through a dorsal approach.
Utilises an osteotomy at the base of the distal phalanx.
Utilises an osteotomy which is transverse in nature.

References

1: Boberg JS, Menn JJ, Brown WL.


The distal akin osteotomy: a new approach.
J Foot Surg. 1991 Sep-Oct;30(5):431-6.

2: Springer KR.
The role of the akin osteotomy in the surgical management of hallux abducto-valgus.
Clin Podiatr Med Surg. 1989 Jan;6(1):115-31.

Question 65
All of the following therapeutic agents used in the management of metastatic disease
of bone act by inhibiting the proliferation of tumour cells EXCEPT
Tamoxifen
Iodine-131
Methotrexate
Pamidronate
Anastrazole

References

Ref 1.Oxford Textbook of Orthopaedics and Trauma Volume 1 p 340-347


Ref 2. Review of Orthopaedics, Miller 4th Ed. P 498-500
Ref 3. Savage P and Ward W. Medical Management of Metastatic Skeletal Disease.
Orthopaedic Clinics of North America. Vol 31.4, Oct 2000, p 545-555
Pamidronate targets areas of bone where the osteoclast activity is high. It helps to bring the
balance of osteoclast and osteoblast activity back to normal. This can reduce pain and help to
strengthen the bone. It also means that less calcium will be lost from the bones.
Tamoxifen is a competitive receptor antagonist of Oestrogen.
Iodine-131 (Radio-iodine) is used against thyroid cancer.
Methotrexate acts specifically during DNA and RNA synthesis, and thus it is cytotoxic during
the S-phase of the cell cycle. Logically, it therefore has a greater toxic effect on rapidly
dividing cells such as malignant cells.
Anastrazole is an aromatase inhibitor which blocks the conversion of androgens to oestrogen,
thus starving Oestrogen sensisitive breast cancers.

Question 66
Poliomyelitis results from a viral infection of which of the following?
Dorsal root ganglion cells
Skeletal muscle cells
Schwann cells
Smooth muscle cells
Anterior horn cells

References

Warner WC. Chapter 31. Paralytic Disorders. Campbell’s Operative Orthopaedics 10th
Edition. Editor S T Canale
Once the poliomyelitis virus invades the body through the oropharyngeal route, it multiplies in
the alimentary tract lymph nodes and then spreads through the blood, acutely attacking the
anterior horn cells of the spinal cord, especially in the lumbar and cervical enlargements. The
anterior horn motor cells may be damaged directly by viral multiplication or toxic byproducts of
the virus or indirectly by ischemia, edema, and hemorrhage in the glial tissues

Question 67
Release of parathyroid hormone in response to hypocalcaemia leads to
Decreased excretion of urinary phosphate.
Decreased conversion of 25-hydroxy-vitamin D to 1,25-dihydroxy-vitamin D
Decreased gut absorption of calcium.
Increased gut absorption of phosphate as a response to increased renal loss
Increased loss of urinary calcium.

References

PTH is secreted in response to low serum calcium. It increases gut absorption of calcium
through stimulating the synthesis of biologically active 1-25 dihydroxy vitamin D3 from 25
hydroxy vitamin D3. PTH also increases reabsorption of calcium from the kidney. but as the
serum calcium level inceases total glomerular filtration of calcium increases, leading to a loss
of calcium from the kidney despite this increased reabsorption.
PTH increases osteocytic osteolysis. The osteocytes transfer calcium from bone canalicular
fluid to interstitial fluid. It also, more slowly, increases osteoclastic reabsorption of mineralised
bone. 4.
PTH prevents the binding of calcium to phosphate by by increasing renal phosphate
excretion. There is a secondary increased gut absorption of phosphate.
Principles of physiology Berne and Levy 2nd ed.

Question 68
Which of the statements below is true for adolescent idiopathic scoliosis
Right thoracic curves are rare and the spinal cord should be evaluated by MRI

A curve magnitude of >20 degrees is a risk factor for progression


The spinous processes swing round towards the convexity of the curve
Curves are commonly associated with an increased thoracic kyphosis
Is more common in males

References
Adolescent idiopathic scoliosis is present in 2 to 4 percent of children between 10 and 16
years of age. It is defined as a lateral curvature of the spine greater than 10 degrees
accompanied by vertebral rotation. It is thought to be a multigene dominant condition with
variable phenotypic expression. Scoliosis can be identified by the Adam's forward bend test
during physical examination. Severe pain, a left thoracic curve or an abnormal neurologic
examination are red flags that point to a secondary cause for the spinal deformity.
Specialty consultation and magnetic resonance imaging are needed if red flags are present.
Of adolescents diagnosed with scoliosis, only 10 percent have curves that progress and
require medical intervention. The main risk factors for curve progression are a large curve
magnitude, skeletal immaturity and female gender. The likelihood of curve progression can be
estimated by measuring the curve magnitude using the Cobb method on radiographs and by
assessing skeletal growth potential using Tanner staging and Risser grading.
Idiopathic scoliosis is lordoscoliotic, resulting in a hypokyphosis.
(Am Fam Physician 2001;64:111-6.) Review of Orthopaedics. Ed Miller. 4th Edition p173-6
Apley's System of Orthopaedics. 8th Edition. p 374-82

Question 69
When comparing in situ decompression with intramuscular transposition of ulnar
nerve for compressive neuropathy which of the following statements is most correct?
Immediate postoperative immobilisation is always necessary following intramuscular
transposition
Clinical outcomes from in situ ulnar nerve decompression are more favourable
Intramuscular transposition is recommended surgical technique for revision cases of failed in
situ ulnar nerve decompression.
Intramuscular transposition commonly results in a weakened flexor-pronator muscle mass.

In situ decompression is recommended for the throwing athlete

References

Millers, J Am Acad Orthop Surg 2007;15:672-81 Entrapment neuropathy of the ulnar nerve.
Elhassan B & Steinmenn SP
Generally, any surgical technique yields 80-90% good results when symptoms are intermittent
and denervation has not occurred. Better results with fewer recurrences are seen with
anterior submuscular transposition in cases with moderate (continuous) symptoms and
severe (evidence of denervation) compression. Poor prognosis correlates with intrinsic
muscle atrophy

Question 70
In the Weaver Dunn procedure, which soft tissue structure is used to stabilise the
distal clavicle?
Acromio-clavicular ligament
Coraco-acromial ligament
Coraco-clavicular ligament
Coraco-humeral ligament
Gleno-humeral ligament

References

Treatment of acromioclavicular injuries, especially complete acromioclavicular separation.


Weaver JK, Dunn HK. J Bone Joint Surg Am. 1972;54:1187-1194

Question 71
In the brachial plexus, which nerve arises from the upper trunk other than the nerve to
subclavius?
Suprascapular nerve
Thoracodorsal nerve
Axillary nerve
Upper subscapular nerve
Long thoracic nerve

References

Any standard anatomy textbook

Question 72
Which of the following is not associated with a higher incidence of Dupuytren's
contracture
Diabetes Mellitus
Female sex
Epilepsy
Scandinavian or Celtic origin
Alcoholism

References

The incidence of Dupuytren contracture is highest in Caucasians, historically those of Celtic


descent. It affects men 7-15 times more often than it does women. More common above the
age of 50 years. It is transmitted in an autosomal dominant fashion with variable penetrance.
It is nearly 3 times more common in those who express HLA -DR3. Other associations are
diabetes, smoking, chronic alcoholism, seizures, and infection. Younger individuals with a
positive family history other fibromatoses is called Dupuytren's diathesis and is associated
with higher recurrence rate. One cause may be localized ischemia and subsequent xanthine
oxidase–derived free-radical formation from endothelial cells. Fibroblasts proliferate within the
fascia, clustered around the microvasculature. Experimentally lower concentrations of free
radicals cause fibroblast proliferation in laboratory cultures. Because active fibroblasts
produce free radicals as well, the fibroblasts induce an autocrine positive-feedback effect on
themselves, causing further ischemia to the microvasculature. The fibroblast growth factor
(FGF), platelet-derived growth factor (PDGF), and transforming growth factor-beta (TGF-?)
may signal the overproduction of the myofibroblasts and/or myofibroblastic activity of the
fibroblasts. In addition, high levels of TGF?? may hinder apoptosis, or cell death, of the active
myofibroblasts, unlike normal tissue healing. The increased concentration and activity of the
myofibroblasts not only increase the total amount of collagen leading to the pathologic nodule
but also cause remodeling of the normal collagen matrix and an increase in the ratio of type I
collagen to type III collagen. The most radical surgery is dermofasciectomy and has lowest
risk of recurrence. The most common surgical procedure performed is partial fasciectomy.

Question 73
A patient presenting with Slipped Upper Femoral Epiphysis
Is unlikely to be going through the pubertal growth spurt
Is less likely to develop avascular necrosis of the femoral head if closed anatomical reduction
is achieved at operation.
Gives a history of trauma in less than half of cases.
Could develop avascular necrosis of the femoral head if weight bearing at presentation
Under the age of 10 may have an underlying endocrine abnormality

References

In SUFE Male age at presentation: 12-14 years Female age at presentation: 10-13 years
Most patients have a relatively uniform skeletal age i.e. the young are advanced and the older
children are borderline immature. The slip appears to occur in a narrow skeletal age range. In
girls SCFE almost exclusively occurs before the menarche. Predisposing factors: obesity,
rapid growth, endocrinopathies such as hypothyroidism, renal rickets, pituitary deficiency, GH
deficiency when treated with GH. Classification: On presentation as Acute (3 week), Chronic
(>3 week), acute on chronic. Loader classification: Stable and unstable based on weight
bearing status. Severity based on Southwick Angle (head-shaft angle in lateral view)- 1:
30degrees, 2: 30 to 60 degrees, 3: >60 degrees Review of Orthopaedics, Miller (Saunders)
Apley's System of Orthopaedics, Solomon (Arnold)

Question 74
The following statement regarding Slipped Upper Femoral Epiphysis is FALSE
Occurs more frequently in boys
Premature physeal fusion usually occurs within two years
The slip occurs between the zone of proliferation and the zone of maturation

On antero-posterior plain radiograhs, Trethowan's line passes superior to the head.


The neck slips anteriorly in relation to the head.

References

In SUFE It is more common in boys and on the left side. Once slipped physis often closes
early. It occurs through the zone of hypertrophy, which is between the proliferation and
maturation. Trethowan's line also known as Klein's line passes above the head of femur in AP
view. The neck of femur slips anteriorly and superiorly in repation to the head. 1. Appley's
System of Orthopaedics (Arnold) 2. Review of Orthopaedics - Miller (Saunders)

Question 75
During ilioinguinal approach, actively bleeding vascular bundle crossing the superior
pubic ramus is encountered. The artery is most likely an anastomosis between which
of the following arteries?
Superior epigastric and common femoral.
Superficial femoral and profunda femoris.
Inferior gluteal and obturator.
External iliac and internal iliac.
External iliac and obturator.

Question 76
The predominant collagen in the knee menisci is
Type I
Type II
Type III
Type IV
Type V

References

Collagen sites Type 1 - bone, tendon, meniscus, annulus of intervertebral disc, skin 2 -
Articular cartilage, Nucleus propulsus 3 - Skin, Blood vessel 4 - Basement Membrane 5 -
Articular cartilage (small amounts) 6 - Thethers condrocytes to the ECM in articular cartilage 7
- Epithelial Basement Membrane 8 - Epithelial basement membrane 9 - Articular Cartilage
(small amounts) 10 - Hypertrophic cartilage 11 - Articular Cartilage (small amounts) 12 -
Tendon 13 - Endothelial cells

Question 77
In which one of the following scenarios could you most confidently attempt irrigation
and debridement of an infected total joint arthroplasty with salvage of the prosthesis
If the infecting organism is Staph Epidermidis
The prescence of a glycocalyx biofilm
Acute infection within 2-3 weeks of arthroplasty
Acute infection more than 6 months following arthroplasty
If the infecting organism is Staph Aureus

References

Early acute infection of prosthetic joint can be salvaged by debridement and washout followed
by high dose IV and then oral antibiotic treatment of appropriate sensitivity. For hip and knee
replacement success rate of more than up to 70% quoted in a recent publications -Toms et al
JBJS Br 2006, Phillips et al JBJS Br 2006 Campbell's Operative Orthopaedics Miller - Review
of Orthopaedics

Question 78
Which of the following interventions has been demonstrated to definitely reduce deep
sepsis following joint replacement?
Double gloving
Antibiotic prophylaxis before invasive dental procedures
Use of wound lavage
Ultraclean air theatres
Use of a hood over the head

References

Ref 1. Tanner J, Parkinson H. Double gloving to reduce surgical cross-infection. Cochrane


Database Syst Rev. 2006 Jul 19;3: CD003087. Review.

Ref 2: Uçkay I et al. Antibiotic prophylaxis before invasive dental procedures in patients with
arthroplasties of the hip and knee. J Bone Joint Surg Br. 2008 Jul;90(7):833-8.
Ref 3. Lidwell et al. Effect of ultraclean air in operating rooms on deep sepsis in the joint after
total hip or knee replacement: a randomised study. Br Med J (Clin Res Ed). 1982 July 3;
285(6334): 10–14.
Ref 1. There is no direct evidence that additional glove protection worn by the surgical team
reduces surgical site infections in patients.
Ref 2. A review of published literature to to determine the evidence for routine antibiotic
prophylaxis prior to a dental procedure was found to lack evidence-based information.
Ref 3. Analysis of 8052 joint replacements showed deep sepsis rate of 1.5% in the group
operated in conventionally ventilated room and 0.6% in the ultraclean-air group (p < 0.001).
There is no evidence to support the use of wound lavage or caps to cover hair but it is
established best practice

Question 79
Which of the following grafts / substitutes has the best osteoconductive properties?
Calcium Hydroxyapatite
Demineralised bone matrix
Cortical allograft
Cancellous autograft
Cortical autograft

References

Review of Orthopaedics, Miller (4th Edition, p20)


Osteoconduction; ‘acts as a scaffold or framework into which bone growth occurs’

Question 80
What would be important part of your surgical management of a infection of a Total
Ankle Arthroplasty presenting after 1 year of implantation with a sinus in front of the
ankle joint. Select the most correct answer.
A. Exchange of Polyethylene Component and high dose antibiotics
B. Removal of entire prosthesis and cement
C. Thorough irrigation and debridement
D. Aspiration of the joint and high dose antibiotics according to sensitivity
E. B and C
References

Principles of prosthetic joint infection are

1 Diagnosis
2 Debridement, irrigation and removal of implant
3 High dose antibiotic IV followed by oral antibiotics and inflammatory marker monitoring
4 Once joint infection is cleared further reconstruction

Campbell's Operative Orthopaedics


Miller- Review of Orthopaedics

Question 81
Which one of following statements is FALSE in the bone metabolism ?
Osteoblasts contain receptors for parathyroid hormone, sex steroids, glucocorticoids, vitamin
D3, insulin and thyroid hormone.
Alkaline phosphatase is a hallmark of osteoblast cells and is increased in serum at times of
increased bone formation.
Serum calcium levels are below normal in patients with osteoporosis
Activated vitamin D regulates the transport of calcium intake from the intestine.
Loop diuretics like frusemide impair resorption of calcium in the thick ascending loop of Henle
in the kidney.

References

OKU Home study syllabus 8

Question 82
The most appropriate candidate for undergoing a reverse polarity shoulder
replacement is
A 50 year old manual worker with acute rotator cuff tear.
A 65 year old rheumatoid patient with painful right shoulder.
A 70 year old patient with painful shoulder due to cuff arthropathy.
A 65 year old patient with painful right shoulder due to osteoarthritis with an intact rotator
cuff.
A 70 year old patient with painful right shoulder arthrosis following traumatic injury to the
shoulder with resultant axillary nerve palsy.

References

Boulahia A, Edwards TB, Walch G. Baratta RV (2002). early results of a reverse design
prosthesis in the treatment of arthritis of the shoulder in elderly patients with a large rotator
cuff tear. Orthopaedics 25; 129-133.

Question 83
Which of the following clinical and radiological features is NOT consistent with static
reducible scapholunate dissociation?
A palpable clunk on Kirk Watson's test
A scapholunate angle of 50 degrees on a lateral radiograph
A scapholunate interval of 4mm on the AP radiograph
An apparent foreshortening of the scaphoid on the AP radiograph
An extended posture of the lunate on radiographs

References

Green’s Operative Hand Surgery.5th Edition pp.555-556.


Scapholunate dissociation is the most common carpal instability. Scapholunate instability is
associated with increased scaphoid flexion and pronation with associated lunate extension.
The abnormal kinematics leads to a decrease in surface area contact at the radioscaphoid
joint. This abnormal articulation causes an increased concentration of load, leading to the
development of degenerative arthritis. The patient may give a history of clicking and clunking
of the wrist. On examination, there is tenderness about the scapholunate interval, which lies
just distal to the Lister tubercle. Provocative manoeuvres for scapholunate instability, such as
the scaphoid shift test, may be positive and there is often associated grip strength weakness.
Radiographs show in AP view "Terry Thomas"sign gap between scaphoid and lunate normal
1-2 mm (abnormal 3mm) Progressive flexion and foreshortening of the scaphoid leads to the
scaphoid ring sign, seeing scaphoid end on. The normal lateral scapholunate angle is 45-60
degrees.

Question 84
Which of the following structures is the most common origin for a volar ganglion in the
hand and wrist?
The scapholunate ligament
The triangular fibrocartilage complex
The flexor retinaculum
The distal radioulnar joint capsule
The scaphotrapezial joint capsule

References

Answer E The scaphotrapezial joint capsule

The most common hand/wrist ganglion is the dorsal wrist ganglion typically arising from the
scapholunate ligament.

The second most common hand/wrist ganglion is the volar wrist ganglion, typically arising
from either the radiocarpal ligament or scaphotrapezial joint capsule. Radiocarpal ligament is
not an option so answer is scaphotrapezial joint capsule.

Question 85
In carrying out an open bone biopsy, all of the following are established good practice
for carrying out the procedure EXCEPT
Making a longitudinal incision where possible
Avoiding the use of staples
Making the biopsy incision in the line of the intended approach for definitive surgery
Avoid crossing compartments where possible
Placing a drain at least five centimetres to the side of the incision

References

Bonetumours.org, Orthoteers, Wheeless textbook of orthopaedics


Review of Orthopaedics, Miller (4th Edition, p442-443)
Complications of poorly planned biopsies include errors in diagnosis, non diagnostic biopsy
material, infection, haematoma, and contamination of nearby tissues. It has been shown that
biopsy complications may lead to an otherwise avoidable amputation. Biopsy should be made
through adequate longitudinal incisions that approach the lesion most directly. Since the
compartments of the foot are numerous, small, and incompletely bounded, it may be difficult
to approach a lesion surgically without contaminating nearby normal tissues. Contamination
of the neurovascular bundle near the medial malleolus is particularly problematic and may
contribute to an unnecessary amputation. It is essential to prevent unintentional spread of the
tumour due to post biopsy bleeding. The tourniquet should be released after the tumour is
sampled, and measures should be taken to insure that the wound is completely dry. A
moderate compressive dressing should be applied and the patient is made non-weight
bearing with the extremity elevated for 3 or 4 days to prevent formation of a hematoma that
may carry tumour cells and track under subcutaneous tissues or through intramuscular
spaces, causing a wide zone of contaminated tissues. Needle biopsy should be performed in
centres where dedicated teams of surgeons and pathologists have perfected the technique,
or for lesions where there is little doubt about the diagnosis and the biopsy is only needed to
confirm what is already known.

Question 86
X-rays of the forearm of a 30 year old man after a fall reveal a fracture of the proximal
third of the ulna and a postero-lateral dislocation of radial head. The patient complains
of weakness in the hand. Clinical examination is most likely to reveal
Weakness of power pinch.
Weakness of flexion of the inter-phalangeal joint of the thumb
Mild clawing of the ulnar digits
Extension of the wrist in radial deviation
An inability to oppose the thumb

References

OKU 5 - pages 275-276, Handbook of Fractures Third edition. Pages 219 -222 and
548.Monteggia fracture-dislocation. Bado classified into four types depending on the direction
of displacement. Also Monteggia fracture equivalents in children for each of the adult types.

Question 87
What is the most important factor in the stimulation of macrophage response to
particulate debris?
Particulate size
Particulate shape
Particulate material
Joint involved
Previous particulate synovitis

References
Maloney W, Lane, S
Periprosthetic Osteolysis in Total Hip Arthroplasty: the Role of Particulate Wear Debris
J. Bone Joint Surg. Am., Sep 1995; 77: 1448 - 1461.
Gruen TA, McNeice GM, Amstutz HC. Modes of failure of cemented stem-type femoral
components: a radiographic analysis of loosening.
Clin Orthop 1979 Jun;(141):17-27

H. -G. WILLERT, J. LUDWIG, and M. SEMLITSCH


Reaction of Bone to Methacrylate after Hip Arthroplasty: A LONG-TERM GROSS, LIGHT
MICROSCOPIC, AND SCANNING ELECTRON MICROSCOPIC STUDY
J. Bone Joint Surg. Am., Oct 1974; 56: 1368 - 1382.

Miller, Orthoteers.
Any particle can serve as a source of wear debris and cause osteolysis. Submicron particles
that are irregularly shaped are known to be highly biologically active. Because of the volume
PE generated, it is the major source of wear particles and osteolysis. The final common
pathway in osteolysis is the release of mediators from osteoclasts that digest bone. Therefore
bisphonates are being tried to reduce osteolysis. Genetic predisposition is also thought to be
significant.

Question 88
Reverse oblique fractures fixed with sliding screw and plate fixation is likely to fail due
to
Screw cut out
Fracture of the plate
Medialisation of the shaft
Communition
Malunion

References

Kenneth Koval, Joseph Zuckerman. Handbook of Fractures, Third edition. Pages 329 – 337.
In a reverse obliquity fracture, the fracture line extends from the medial cortex proximally to
distal lateral cortex. It is inherently unstable because of the tendency for medial displacement
of the femoral shaft.

Question 89
Which vessel arises from the second part of the right subclavian artery?
Internal thoracic artery
Dorsal scapular artery
Vertebral artery
Costocervical trunk
Thyrocervical trunk

References

Last’s anatomy Regional and Applied. Tenth edition. CS Sinnatamby. Page 340
The only branch of the second part of subclavian artery is the costocervical trunk.

Question 90
Which is the most accurate way to diagnose pulmonary embolism ?
Pleuritic pain, tachypnea and tachycardia
Right bundle branch block, right axis deviation and ST depression in lead III on ECG
Nuclear medicine ventilation-perfusion (V/Q) scan
Contrast angiography
Collapse lung on chest radiography

References

Predisposing factors for thromboembolism

General - Previous history of DVT


Aging
Use of general anaesthetic c/w spinal anaesthetic
Virchow's triad- stasis, hypercoagulability and intimal damage

Factors affecting venous stasis are -Immobility Paralysis Pregnancy Varicose veins Obesity
Congestive cardiac failure Recent MI Infection Inflammatory Bowel disease

Factors causing hypercoagulability are -Malignancy COCP Increased blood viscosity Smoking
Clotting abnormalities, AT III def, Factor V Leiden, Homocysteinuria, Sickle cell,
Polycythaemia, Thrombocythaemia

Factors causing intimal injury - Trauma

Low Risk -
Minor Surgery (<30mins); no other risk factors other than age
Major Surgery (>30mins); 40 yrs; no other risk factors
Minor Trauma or medical illness

Moderate Risk
Major Surgery; >40 yrs or other risk factors
Major medical illness eg Ca, Cardiac, Pulmonary
Major Trauma or Burns
Minor Surgery with past thrombo/embolic event

High Risk
Fracture or Major Orthopaedic Surgery to Pelvis/Lower Limb
Major Pelvic or Abdominal Surgery for Ca
Major Surgery, Trauma or illness with past thrombo/embolic event

Gold standard investigation - pulmonary angiography.

OWLS online
Miller. Review of orthopaedics, third edition. Basic Science, page 107-8

Question 91
With regard to the Brown-Sequard syndrome all the statements are true EXCEPT
It causes ipsilateral motor loss
It causes contralateral propioception loss
It is usually the result of penetrating trauma
It has the best prognosis of all the incomplete spinal cord lesions
It causes contralateral pain and temperature loss

References

Brown-Séquard syndrome is an incomplete spinal cord lesion characterized by a clinical


picture reflecting hemisection of the spinal cord, often in the cervical cord region. It was first
described in the 1840s after Dr. Charles Edouard Brown-Sequard sectioned half of the spinal
cord. It is a rare syndrome, consisting of ipsilateral hemiplegia with contralateral pain and
temperature sensation deficits because of the crossing of the fibers of the spinothalamic tract.
Brown-Sequard syndrome is usually the result of penetrating trauma to the cervical or
thoracic spine. It is now also associated rarely with herniated cervical discs.

Question 92
Opposition of the thumb involves all of the following except
Movement of the thumb towards the fingers
Abduction of the thumb from the palmer surface of the index finger
Radial deviation of the proximal phalanx of the thumb on the metacarpal
External rotation and supination of the thumb
Flexion of the metacarpo-phalangeal joint of the thumb

References

Campbells Operative Orthopaedics


Opposition of thumb and its restoration by Jacobs et al JBJS Am 1960

Question 93
The condition most commonly associated with latex allergy is
Cerebral Palsy.
Spina Bifida
Talipes Equinovarus
Achondroplasia
Morquios syndrome.

References

JB Emans. Allergy to latex in patients who have myelodysplasia. Relevance for the
orthopaedic surgeon
J. Bone Joint Surg. Am., Aug 1992; 74: 1103 - 1109.
Latex allergy in myelodysplasia was first reported in 1984. Life threatening IgE mediated
immune reactions, including intra-operative anaphylactic reactions, have been reported in
spina bifida and congenital urethral anamolies (use of barium tipped catheters). Sensitisation
is presumably due to repeated catheterisation or multiple operations in these patients. It is
mandatory that patients are enquired about latex allergy and latex-free environment is
provided.

Question 94
Which of the following statements is TRUE for osteonecrosis following a talar neck
fracture:
Due to the high incidence of osteonecrosis following fracture dislocations, primary triple
arthrodesis is the treatment of choice in this fracture pattern
Predominant blood supply to the talus comes from branches of peroneal artery.
Revascularisation of the talus can take up to three years
Nondisplaced fractures have 30% incidence of avascular necrosis of talus.
Hawkins's sign suggests failure of talar body revascularisation

References

Avascular necrosis
Hawkins 1: 0- 13%
Hawkins 2: 20- 50%
Hawkins 3: 20-100%
Hawkins sign provides evidence of revascularization of talar body as indicated by patchy
subchondral osteoporosis at approx 6-8 weeks indicates that there is sufficient vascular
supply to bone to allow normal disuse osteopenia to occur and that normal healing is
occuring.

Incidence of arthritis - 40-90% is largely related to articular damage, subchondral collapse


(from AVN), immobilization, and malunion. Delayed union (more than 6 months) is common.

Principles of treatment are emergent ORIF all open/unreducible fractures. Surgeon should
attempt reconstruction and avoid arthrodesis. It is recommended to use rigid, interfragmentary
compression screws (3.5, 4.0, 6.5 mm). The aim of fracture treatment with talar neck fractures
is to restore neck to its anatomic position and avoid varus or supination malalignment of the
talar neck.

Bucholz R.W., Heckman J.D., Court-Brown C., Tornetta P., Koval K.J., Rockwood and
Green's Fractures in Adults; Lippencott, Williams and Wilkins, Philidelphia, 2005: 2091-2108

Question 95
Which of the following is NOT an ECG change commonly associated with pulmonary
embolism?
Tall T waves in lead III
Sinus tachycardia
Right bundle branch block
Q waves in lead III
Atrial fibrillation

References

Oxford textbook of medicine


Rodger M, et al. Diagnostic value of the electrocardiogram in suspected pulmonary embolism.
Am J Cardiol October 1, 2000;86:807-9.
Question Comments
Rodger and associates evaluated all patients with suspected PE who were seen at a large
tertiary-care hospital and referred for ventilation-perfusion scanning or pulmonary
angiography. The referring physician first assigned an index of clinical suspicion for PE based
on all available clinical data, including the history, physical examination, blood gases, ECG, d-
dimers and chest radiograph. All patients then underwent ventilation-perfusion scanning. After
the scan, patients with a posttest probability of PE of less than 5 percent and a low index of
pretest clinical suspicion were considered not to have PE. Patients with a posttest probability
of greater than 88 percent and an intermediate or high index of pretest clinical suspicion were
considered to have PE. All other patients were recommended to proceed to pulmonary
angiography, which some did, depending on the patient's treating physician. Patients with
indeterminate scores who did not have angiography were excluded from the analysis.
Standard 12-lead ECGs were obtained on 189 of the 212 classified patients (PE or no PE)
and analysed for 28 features thought to be more common with PE. Only tachycardia and
incomplete right bundle branch block were significantly more frequent in patients with PE than
those without PE. S1Q3T3 was equally likely to be found in patients without PE who were
initially suspected to have PE. Previous investigators found other ECG changes to be
significantly more common in patients with PE, looked only at specific patient populations or
used inadequate diagnostic criteria to identify patients with PE.
The authors conclude that the ECG is of limited diagnostic value in patients with suspected
PE. Many of the classically described ECG changes in patients with suspected PE are equally
common in patients suspected of having PE but in whom the diagnosis is ultimately excluded.
Even the two statistically significant ECG changes noted in this study, tachycardia and
incomplete right bundle branch block, are rarely observed and are only slightly more frequent
in patients with PE.

Question 96
Tendo Achilles lengthening is required most often in which of the following
conditions?
Duchenne muscular dystrophy
Idiopathic toe walker
Cerebral palsy
Spinal muscular atrophy
Charcot Marie Tooth disease

References

Miller orthopaedics
Cerebral palsy is a non-progressive neuromuscular disorder resulting from injury to the
immature brain. Note that the neurological lesion is non-progressive but the deformities are
progressive. Spasticity in the gastrocnemius- soleus complex results in equines contracture of
the ankle. The Silverskoild differentiates between gastrocnemius/soleus tightness.

Question 97
Amputation of the 5th toe
Reduces walking speed by 20%
Reduces running speed by 30%
It has no obvious effects on walking/ standing
Usually results in a clinical limp
It is less well tolerated if performed with excision of lateral condyle of the metatarsal head.

References
Most common indications are
Infection (wet gangrene) in diabetic patients with neuropathy
Dry gangrene due to vascular insufficiency in diabetes, collagen diseases.
Trauma, eg. Lawnmower injuries, motorcycle injuries, frostbite, burns
Tumour
Surgical principles in foot amputation/disarticulation

Amputation is a reconstructive procedure, not a failure


Eliminate life threatening disease
Create a mobile soft tissue envelope with proper contouring of cut bones to absorb shear and
direct forces to make the interface with the prosthesis comfortable. Ideally formed by plantar
skin, subcut tissue and investing fascia. Avoid split skin grafting on distal, lateral and plantar
surfaces in adults as ulcers may occur. Split skin grafting suitable for dorsal surface.
Prevent equinus contracture of the ankle joint in all transverse ablations proximal to MTP
joints, by casting foot in slight dorsiflexion.

5 th toe, can leave metatarsal head prominent, therefore trim its lateral condyle. 1st & 5th ray
excisions function well in normal shoes.

Campbell's operative orthopaedics.

Question 98
For which of the following skeletal tumours is radiation therapy routinely used for
definitive local control?
Osteogenic sarcoma
Ewing's sarcoma
Chondrosarcoma
Pleomorphic sarcoma
Paget's sarcoma

References

Basic Orthopaedic Sciences (Ramanchandran)


Review of Orthopaedics (Miller)
Osteogenic sarcoma is a high grade spindle cell tumour with a bimodal age distribution. It
occurs most commonly in the knee (50%) and humerus (25%). Radiological features include
Sclerosis, permeative destruction and presence of Codman’s Triangle. It is managed with
resection then adjuvant chemotherapy.
Ewing’s sarcomas are small round cell tumours of uncertain origin. They are caused by a
genen translocation 80% occurring in the first two decades. Commonly occur in femoral
diaphysis with M:F of 3:2. They have an onion skin appearance due to periosteal reaction and
new bone formation and a large soft tissue element. The mainstay of management is with
neoadjuvant chemotherapy and DXT
Chondrosarcoma is a cartilage malignancy occurring in middle age. Patchy calcification gives
a popcorn appearance. They have a narrow zone of transition and may fool observers into
thinking they are benign. Surgical resection forms the mainstay of treatment.
Pleomorphic sarcoma is rare and occurs in adults >60. Occurs in deep soft tissues of lower
lmbs as well as those that are well of de differentiated being retroperitoneal. They are locally
invasive, exhibit nuclear atypia and are managed by excision if feasible.
Pagets sarcoma occurs rarely on a background of paget’s disease of bone. This is
characterised by disordered bone remodelling with broad trabecullae reversal of cement lines.
It can be monostotic of polyostotic. Sarcoma should be suspected were increased pain
swelling and cortical and soft tissue destruction are present. These tumours are mostly
managed palliatively due to poor prognosis and <20% 5 year survival.
Question 99
How should an open biopsy of bone for suspected bone tumour carried out?
By the surgeon likely to be responsible for any further surgery for the tumour at a specialist
centre
By the first surgeon to suspect a neoplastic process at his local institution
Prior to any imaging of the lesion
Always by carrying out an excision biopsy for any tumour under 5cm in diameter
Initially by blind percutaneous passage of a Jamshidi needle

References

Orthoteers, Oxford Textbook of Orthopaedics


The surgeon should know the probable diagnosis and stage of tumour before biopsy as it is
the last step in the staging of the patient. The decision for surgery should be made after
consultation with the pathologist and radiologist (MDM).
The biopsy should be performed by the surgeon who will perform the definitive surgery.
Biopsy tract orientation & location are critical - as this will need to be included in the definitive
surgery if the lesion is malignant. Needle biopsy is less sensitive and less specific.
Open Biopsy: Aim for an excisional biopsy when possible especially in benign lesions.
Incisional biopsy is also preferable in malignant lesions. After a longitudinal incision sharp
dissection should proceed directly to the tumour, through muscle not between muscle planes.
Uninvolved anatomic compartments should not be exposed. Avoid all major neurovascular
structures to prevent contamination. Excise block of reactive tissue, pseudo capsule, capsule,
and block of tumour, send for histology in formalin or as a frozen section. Samples should
also be sent for microbiological analysis.
Windows in bone should be as small as possible and oval to avoid stress risers and
pathological fracture. Release the tourniquet prior to closure, meticulous haemostasis is
required to avoid tracking haematomas. Close with a subcuticular. Stitch. Drains should come
out through the wound.
If proceeding following the biopsy -> new instruments and drapes should be used to prevent
seeding.
The Jamshidi needle is routinely used for bone marrow aspiration. It can be used for bone
tumour biopsy but this should be done under imaging at a specialist centre.

Question 100
A 74 year old man is having prophylactic nailing of an isolated proximal femoral
metastasis from a renal cell carcinoma. He also has atrial fibrillation for which he takes
warfarin. The potential for intraoperative blood loss has been raised. Based on the
information above, which measure would be CONTRAINDICATED?
Intraoperative use of a cell saver for autologous transfusion
Preoperative haemoglobin check and transfusion if low
Cessation of warfarin a week preoperatively
Preoperative embolisation
Cross-matching at least 4 units of packed red cells to be available at the time of operation

References

Department of Health "Better Blood Transfusion Toolkit

Question 101
Clostridium difficile colitis is most commonly associated with which of the following
antibiotics:
Oral Vancomycin
Single dose gentamicin
Penicillin G
Oral metronidazole
Clindamycin

References

British National Formulary and online literature search.


C.diff colitis is most common when broad spectrum antibiotics are given over a long period of
time. It can also be caused by long term administration of penicillin G however this is much
less frequent than with clindamycin or clarithromycin.

Question 102
Which of the following is NOT a pro-thrombogenic condition that may require special
consideration for thromboprophylaxis in orthopaedic surgery
Factor V Leiden
Christmas Disease
Protein S deficiency
Protein C deficiency
Antithrombin III deficiency

References

Haemophilia B - Christmas disease is a hereditary blood coagulation disorder. It is caused by


a deficiency of a blood plasma protein called factor IX.

Risk factors for venous thromboembolism


Exponential increase in risk with age. In the general population:
< 40 years - annual risk 1/10,000
60-69 years - annual risk 1/1,000
> 80 years - annual risk 1/100
May reflect immobility and coagulation activation

Obesity
3 x risk if obese (body mass index >= 30 kg/m2)
May reflect immobility and coagulation activation

Varicose veins
1.5 x risk after major general / orthopaedic surgery
But low risk after varicose vein surgery

Previous VTE
Recurrence rate 5% / year, increased by surgery

Thrombophilias
Low coagulation inhibitors (antithrombin, protein C or S)
Activated protein C resistance (e.g. factor V Leiden)
High coagulation factors (I, II, VIII, IX, XI)
Antiphospholipid syndrome
High homocysteine

Other thrombotic states


Malignancy 7 x risk in the general population
Heart failure
Recent myocardial infarction / stroke
Severe infection
Inflammatory bowel disease, nephrotic syndrome
Polycythaemia, paraproteinaemia
Bechet’s disease, paroxysmal nocturnal haemoglobinuria

Hormone therapy
Oral combined contraceptives, HRT, raloxifene, tamoxifen63,64 3 x risk
High-dose progestogens 6 x risk

Pregnancy, puerperium 10 x risk

Immobility
Bedrest > 3 days, plaster cast, paralysis 10 x risk; increases with duration

Prolonged travel

Hospitalisation
Acute trauma, acute illness, surgery 10 x risk

Anaesthesia
2 x general vs spinal / epidural

Question 103
In relation to the anatomy of the posterolateral corner of the knee, Seebacher desribed
three layers. Which one of the following structures is present in layer II?
Ilio-tibial tract
Arcuate ligament
Fabello-fibular ligament
Patella-retinaculum
Biceps tendon

References

A cadaveric study described three distinct layers of the lateral structures of the knee. The
most superficial layer consists of the iliotibial tract and biceps. The second, or middle layer,
consists of the quadriceps retinaculum anteriorly and two patellofemoral ligaments posteriorly.
The third, or deep layer, consists of a superficial and deep capsular lamina. The superficial
lamina includes the LCL and fabellofibular ligament. The deep lamina consists of the coronary
ligament, popliteus hiatus, arcuate ligament, and PFL.

Seebacher JR, Inglis AE, Marshall JL, Warren RF. The structure of
the posterolateral aspect of the knee. J Bone Joint Surg (Am)
1982;64A:536--541.

Miller

Question 104
The distal humerus lateral condyle secondary ossification centre most commonly
appears at
2- 4 years
4-6 years
6-8 years
9-11 years
13-15 years

References

Inuries of childrens elbow are frequently missed and are radiologically difficult to diagnose.
Following is the normal order of ossification centres appearing in distal humerus.

Capitellum 1-3yrs

Medial epicondyle 3-5yrs

Trochlear 5-7yrs

Olecranon 7-9yrs

Lateral epicondyle 9-11yrs

Fitzgerald RH, Kaufer H, Malkani AL; Orthopaedics. Mosby 2002 Page 486

Question 105
Which of the following statements is TRUE about the anatomy of the talus
The talus has tendon insertion
The origin of extensor digitorum brevis is from the talus
A branch of the anterior tibial artery forms the artery to the tarsal canal
The posterior segment of the deltoid ligament attaches to the lateral tubercle
60% of the surface of the talus is articular cartilage

References

The dome, or body of the talus, articulates with the tibia and fibula on its superior, medial, and
lateral surfaces to form the ankle joint. The transverse diameter of the body is greater
anteriorly than posteriorly. This corresponds with the width of the tibial plafond to effect
increased joint stability with dorsiflexion. Inferiorly, the body articulates with the posterior facet
of the calcaneus and together with the underside of the head and neck of the talus forms the
subtalar joint.
The neck projects anteromedially and plantarwards from the dome, and its inferolateral
surface defines the roof of the sinus tarsi and tarsal canal. This portion of the talus marks the
origin of the bifurcate and cervical ligaments. The primary blood supply to the body of the
talus passes under the talar neck as a vascular sling. Severe displacement and comminution
of talar neck fractures may indicate a significant disruption of this crucial blood supply to the
body of the talus.
The head of the talus articulates with the navicular distally and with the calcaneus inferiorly.
Together, the talonavicular joint and the anterior facet of the subtalar joint form the
acetabulum pedis. This joint complex contributes the majority of motion essential to
accommodative hindfoot function.

The talus is vulnerable to avascular necrosis because it relies heavily upon its direct blood
supply. The talus lacks muscular or tendinous insertions and hyaline cartilage covers 60% of
its surface area. This lack of soft-tissue attachments limits the indirect perfusion of the talar
body. Disruption of the artery of the tarsal canal, a branch of the posterior tibial artery, leads
to the loss of the main intraosseous blood supply to the central two-thirds of the talar body.
Anastomoses with the tarsal sinus artery complete the vital vascular sling that travels along
the undersurface of the talar neck. Branches from the anterior tibial and peroneal arteries
supply the talus, but only in rare instances would their contribution to the overall vascularity
sustain adequate perfusion of the body.

Bucholz R.W., Heckman J.D., Court-Brown C., Tornetta P., Koval K.J., Rockwood and
Green's Fractures in Adults; Lippencott, Williams and Wilkins, Philidelphia, 2005: 2091-2108

Question 106
Which of the following conditions affecting the knee joint is NOT likely to cause loose
body formation?
Synovial chondromatosis
Osteochondritis dissecans
Patella osteochondral fracture
Pigmented villonodular synovitis
Osteoarthritis

References

Pigmented villonodular synovitis is a slow growing, benign, and locally invasive tumor of the
synovium.
Location: most often involves the knee (also in hip, ankle, elbow)
Acute episodic attacks of pain and swelling may occur. Patients may have mechanical
symptoms such as locking. Most have hemorrhagic, dark brown synovial fluid.
The biopsy is diagnostic.
Differential diagnosis includes rheumatoid arthritis in which case unlike PVNS, it affects
multiple joints.
Broadly divided sub-types: nodular or a diffuse form

Campbells operative Orthopaedics

Question 107
You are seeing a 5-year-old female child with in-toeing due to excessive femoral
anteversion. She is otherwise normal and healthy, and her mobility is unimpaired. Her
parents are greatly concerned by the cosmetic appearance and possible future
disability, and request that she be treated. You recommend which one of the
following?
Observation
Medial shoe wedges
Torque heels
Sleeping in a Dennis Browne splint for 6 months
Derotational osteotomy of the femur

References

Diets FR: In-toeing: Fact, fiction and opinion. Am Fam Physician 1994;50(6):1249-1259.
Miller: Review of orthopaedics
In toeing is often evident in 3 – 6 yr olds and is due to excessive internal rotation of the femur.
Clinically there in increased internal rotation and decreased external rotation. The patella is
internally rotated. Children sit in the “W” position. Longterm PFJ problems can occur but most
cases resolve by the age of 10. If less than 10 degrees external rotation is present then a
femoral derotational osteotomy is often utilised (best when performed in intertrochanteric
region to allow adequate correction). Medial shoe wedges would worsen internal rotation,
torque heels would not help. Dennis Browne boots are for CTEV and a derotational
osteotomy is best performed at a later date.

Question 108
The integrity of which of the following does Elson's test demonstrate?
The FDP tendon
The central slip of the extensor apparatus
The lateral extensor bands
The lumbrical muscles
The sagittal bands

References

The test described by Elson demonstrates integrity of the central slip - Put finger over edge of
table, with PIPJ flexed to 90deg. and ask the patient to extend against resistance. Weakness
of resisted extension of PIPJ & hyperextension of DIPJ occurs if the central slip is ruptured.

Question 109
Which one of the following statements about Vancouver classification is NOT true ?
Vancouver B2 refers to fractures around the stem with a stable prosthesis and inadequate
bone stock.
Vancouver C fractures distal to femoral stem are most common in cemented implants.
Type B1 fractures should be managed by extramedullary stable fixation with or without bone
grafting.
Type A are subdivided into A(g) and A(l).
Type C can be managed as conventional femoral fractures.
References

Type A fractures are those


situated in the trochanteric region. Type B fractures
are those located around or just distal to the stem. Type
C fractures are those located well below the stem. Type
A fractures are subclassified into Type AG where
fractures involve greater trochanter and Type AL where
fractures involve lesser trochanter. Type B fractures are
subclassified into B1 if the implant is stable, B2 if the
implant is loose and B3 if the implant is loose and bone
stock around the stem is inadequate.

Periprosthetic Fractures following THR:

1% after primary THA


4% after revision THA

Prevention:

Is better than the most succesful treatment option


- avoid Cortical defects
- avoid malalignment of prosthesis
- regular radiological follow-up

Risk factors:

Inadequate calcar cancellous bone removal (with subsequent calcar resorption)


Varus positioning of the stem
Lateral stem nicks produced by drilling for greater trochanteric wires
Progressive osteolysis

MANAGEMENT
Management Aims:
Restore function
United fracture
Stable prosthesis
Preserve bone stock

Management depends on:


Location of fracture
Stability of prosthesis & fracture
Quality of bone stock

Question 110
Which one of the following statements is TRUE for options for primary closure of
wounds
Simple interrupted sutures for contaminated wounds without tension
Simple interrupted sutures for clean wounds without tension
Subcuticular sutures for clean wounds with tension
Vertical mattress sutures for clean wounds with tension
Vertical mattress sutures for contaminated wounds without tension

References
Wound closure techniques have evolved from the earliest development of suturing materials
to comprise resources that include synthetic sutures, absorbables, staples, tapes, and
adhesive compounds. The engineering of sutures in synthetic material along with
standardization of traditional materials (eg, catgut, silk) has made for superior aesthetic
results. Similarly, the creation of natural glues, surgical staples, and tapes to substitute for
sutures has supplemented the armamentarium of wound closure techniques. Aesthetic
closure is based on knowledge of healing mechanisms and skin anatomy as well as on an
appreciation of suture material and closure technique. Choosing the proper materials and
wound closure technique ensures optimal healing.
Three phases of wound healing have been identified and studied on the cellular and
molecular level. These 3 distinct phases are inflammation, tissue formation, and tissue
remodeling.

Closure of wounds should only be undertaken in the abscence of contamination and without
tension on the skin edges.

Essential Surgery
H. Burkitt et al.

Question 111
Which of the following statements is true with regard to sterilisation?
Dry heat sterilisation is more efficient than moist heat sterilisation.
Ethylene oxide can be used to sterilise endoscopes.
Rubber and plastic materials can be sterilised by dry heat sterilisation.
Bacillus subtilis is a chemical indicator used for monitoring sterilisation processes.
Ethylene oxide sterilisation is cheap and safe.

References

Clinical Surgery in General. RCS Course Manual. 2nd edition pgs. 183-184.

Question 112
A 42 year old farmer complains of increasing pain in his great toe metatarso-
phalangeal (MTP) joint. He has had a dorsal cheilectomy 8 years ago. He now has
hallux valgus with a painful range of motion in his 1st MTP joint. The most appropriate
treatment is
1st MTP Moje replacement
Keller's procedure
1st MTP fusion
Akin osteotomy
Scarf osteotomy

References

In a high demand patient, fusion remains the most successful option. Ceramic (Moje)
replacements are not suitable for this group of patients. Keller's procedure leads to a weak
floppy toe with decreased push off strength. Akin and scarf osteotomies are procedures for
hallux valgus and are inappropriate for Hallux rigidus.

Question 113
With regards to 5th metacarpal fractures
A : Scaphoid tubercle
B : AP x-ray of hand
C : Lunate
D : Lateral x-ray of hand
E : 45 degree oblique x-ray of the hand
F : Scaphoid views on x-ray
G : Midline of the wrist
H : Extensor carpi ulnaris
I : Flexor digitorum profundus to little finger
J : Abductor digiti minimi
K : Intrinsic muscles of the hand
1 : During clinical examination of a 5th metacarpal fracture the rotational alignment is
assessed in relation to
Correct answer:
A
Your answer:
A
2 : The major deforming force in a neck of 5th metacarpal fracture is
Correct answer:
K
Your answer:
K
3 : The most relevant radiological investigation in the diagnosis of a 5th carpo-
metacarpal dislocation is
Correct answer:
D
Your answer:
D
4 : The major deforming force for fracture dislocation of 5th metacarpal at the carpo-
metacarpal level is
Correct answer:
H
Your answer:
H

References

Rockwood and Greens fractures in Adults


Green’s operative hand surgery
Rotational alignment can be assessed in reference to the nail folds/ nail orientation, proximal
pole of scaphoid and of the mid point of the wrist following FCR proximally dependent on
which reference text is used. This is therefore ambiguous! The point of assessment depends
upon the level of injury. This stem is ambiguous and should be deleted.
The major deforming force is the intrinsics
The most relevant radiological investigation is a lateral as it allows assessment of angulation
<40 degress and therefore determine operative/ manipulative intervention
Extensor carpi ulnaris is the major deforming force as it inserts solely into the base of the 5th
MC and therefore relies upon a congruent and stable CMCJ.

Question 114
From the following stems please select the most likely diagnosis
A : Medial compartment osteoarthritis
B : Rheumatoid arthritis
C : Osteochondritis dissecans of the knee
D : Tibial plateau fracture
E : Rupture of the anterior cruciate ligament
F : Rupture of the posterior cruciate ligament
G : Degenerative meniscal tear
H : Patello-femoral arthritis
I : Patello-femoral dislocation
J : Dislocation of the knee joint
1 : A 65 year old woman presents with a 12 month history of increasing, poorly
localised anterior knee pain. This limits her mobility especially down stairs. The knee
swells occasionally and has previously 'given way'
Correct answer:
H
Your answer:
H
2 : A 11 year old girl is seen in A+E following a twisting injury sustained whilst playing
netball. She turned to pass the ball and fell to the ground with immediate anterior knee
pain and deformity. The knee quickly became swollen and she has had difficulty weight
bearing. Her mother had history of similar knee problems.
Correct answer:
I
Your answer:
I
3 : A 12 year old basket-ball player injured his knee during the warm up before a match.
He fell to the floor experiencing immediate poorly localised pain. He attempted to start
the game but was substituted after 10 minutes. He notes only minor swelling and
stiffness the following day. He has history of similar problems for the last 1 year.
Correct answer:
C
Your answer:
C

References

Question 115
Regarding upper limb anatomy
A : Short head of biceps
B : Long head of biceps
C : Nerve to subclavius
D : Thoracodorsal nerve
E : Upper subscapular nerve
F : Lower subscapular nerve
G : Pectoralis major
H : Long thoracic nerve
I : Scalenus anterior
J : Lateral pectoral nerve
1 : Which structure is attached to the coracoid process?
Correct answer:
A
Your answer:
A
2 : Injury to which nerve will cause Loss of function of latissimus dorsi muscle?
Correct answer:
D
Your answer:
D
3 : In preganglionic upper trunk brachial plexus injury scapular winging is caused by
loss of which nerve function?
Correct answer:
H
Your answer:
H
References

Last’s Anatomy – Regional and Applied. Tenth edition. C.S Sinnatamby.


Brachial plexus injuries are not common in everyday practice but common in exam situations!
Brachial plexus arises from anterior primary rami of C5,6,7,8 and T1. In a pre-fixed plexus,
contribution from C4 is large and T1 is reduced. In a post-fixed plexus, C5 is always present
and T1 is large. Roots – Trunks – Divisions – Cords – Branches.

Question 116
With regards to the Stress-Strain Curve, please match the definitions to the parameter
described
A : Plastic deformation
B : Creep
C : Viscoelastic material
D : Yield point
E : Ultimate strength
F : Brittle material
G : Fatigue point
H : Ductile material
I : Elastic deformation
J : Breaking point
1 : A material which exhibits stress-strain behaviour that is time and rate dependent
Correct answer:
C
Your answer:
C
2 : A persistent change in length even after the load has been removed
Correct answer:
A
Your answer:
A
3 : The transition point from elastic to plastic behaviour.
Correct answer:
D
Your answer:
D

References

Biomaterials and Biomechanics. Chapter 1; Section 8. page 108. In Review of Orthopaedics.


5th Edition. Editors Miller M D, Hart JA. 2008. Saunders Elsevier.

Question 117
Function of cell
A : Primative Mesenchymal Cell
B : Fibrochondrocyte
C : Eosinophil
D : Synovial Type A
E : Synovial Type B
F : Chondrocyte
G : Osteoclast
H : Osteoblast
I : Platelet
J : Osteocyte
1 : Important in Phagocytosis
Correct answer:
D
Your answer:
A
2 : Target cell for Bone Morphogenic Protein
Correct answer:
A
Your answer:
H
3 : Produces synovial fluid
Correct answer:
E
Your answer:
D
4 : Important for healing meniscal tear
Correct answer:
B
Your answer:
B

References

Mankin, HJ: Current concepts review: The response of articular cartilage to mechanical injury.
JBJS Am 64:460-466
Arnoczky, SP & McDevitt CA. The meniscus:Structure, function, repair, and replacement. In
Buckwalter,JA,Einhorn TA: Orthopaedic Basic Science:Biology and Biomechanics of the
Musculoskeletal System, 2nd ed. pg531-546 Rosemont,IL,American Academy Orth Surg
2000

Question 118
Which type of articulation best describes the following joints
A : Spheroidal
B : Hinge
C : Polyaxial
D : Ellipsoid
E : Saddle
F : Pivot
G : Ball and Socket
H : Gliding
I : Syndesmosis
J : Amphidiarthrodial
1 : The index finger proximal interphalangeal joint
Correct answer:
B
Your answer:
B
2 : The thumb carpo-metacarpal joint
Correct answer:
E
Your answer:
E
3 : The inferior tibiofibular joint
Correct answer:
I
Your answer:
I

Question 119
With regard to bursae around the knee
A : Suprapatellar bursa
B : Infrapatellar bursa
C : Prepatellar bursa
D : Deep patellar bursa
E : Deep to pes anserinus
F : Deep to patellar tendon
G : In the popliteal fossa
H : Lateral to Gerdy’s tubercle
I : Patellar bursa
J : Between the ligaments of Wrisberg and Henry
1 : Which is the bursa most commonly inflamed?
Correct answer:
C
Your answer:
C
2 : Where is the infrapatellar bursa located?
Correct answer:
F
Your answer:
H
3 : Inflammation of which bursa is known as "Housemaid's knee"?
Correct answer:
C
Your answer:
B

References

Wheeless textbook on-line


Gray’s anatomy
There are nine named and un-named bursae around the knee. The named ones are the
prepatellar, infra-patellar, supra-patellar, deep patellar and pes anserine bursae.

Question 120
What is the most appropriate management for the following patients?
A : Physiotherapy stretches
B : Night splinting
C : Conservative management, not for consideration of surgery
D : Conservative management, re-consideration of surgery later
E : Posterior capsulotomy
F : Posteromedial arthroscopic knee release
G : Hamstring release and posterior capsulotomy
H : Posterior capsulotomy
I : Tibial rotational ostetomy
J : Subcutaneous knee flexor tenotomy
1 : A four and a half year old boy with myelomeningocele with knee flexion contracture
who is able to walk
Correct answer:
G
Your answer:
B
2 : A six year old girl with myelomeningocele with knee flexion contracture who is not
ambulant, but can sit stably and transfer
Correct answer:
C
Your answer:
G
3 : According to Rideau what is the management for a four year old boy with Duchenne
muscular dystrophy, showing early contracture of the knee but still ambulant
Correct answer:
J
Your answer:
B

References

1. Marshall PD, Broughton NS, Menelaus MB, et al: Surgical release of knee flexion
contractures in myelomeningocele. J Bone Joint Surg 78B:912-916, 1996 2. Y. Rideau, G.
Duport, A. Delaubier, C. Guillou, A. Renardel Irani and J.R. Bach, Early treatment to preserve
quality of locomotion for children with Duchenne muscular dystrophy. Semin Neurol 15
(1995), pp. 9–17 3. Rideau Y. Treatment of orthopedic deformity during the ambulatory stage
of Duchenne muscular dystrophy. In: Serratrice G, Cros D, Desnuelle C, Gastaut JL, Pellisier
JF, Pouget J, Schiano A, editors. Neuromuscular diseases. New York: Raven Press,
1984:557–564 4. J. Forst, R. Forst. Lower limb surgery in Duchenne muscular dystrophy.
Neuromuscular Disorders, Volume 9, Issue 3, Pages 176-181
Question Comments
In myelomeningocele, "knee flexion contractures may result from spasticity in the hamstring
muscles. In the nonambulatory patient, knee flexion contractures are universal, and do not
usually pose functional problems. In the ambulatory patient, surgical release of the
hamstrings with posterior capsulotomy of the knee joint is occasionally needed and does
improve gait" (Marshall et al, 1996) Finally, it could be proven that neither by daily extensive
physiotherapy nor by prophylactic night splinting the development of lower limb contractures
can be influenced effectively" (Rideau, 1984) Weakness in the hip abductors leads to a
Trendelenburg lurch during gait, with the child leaning the body from side to side over the
stance phase limb. External rotation of the tibia also develops over time in these children.
Biomechanically, weakness and rotational deformity allows for the development of genu
valgum and ligamentous instability of the knee. Correction of tibial rotational deformity with
osteotomy has been proposed to improve gait. (Marshall, 1996) The protocol of early lower
limb surgery after Rideau et al. (1996) includes...a subcutaneous tenotomy of knee flexors
and the lengthening of Achilles’ tendon" (Forst et al, 1999)

Question 121
Regarding incision choice, Langer's lines and wound closure
A : Disregard Langer's lines
B : Z-plasty for incisions crossing Langer's lines
C : S-shaped curvilinear incision across Langers lines
D : Advancement flap
E : Rotation flap
F : Excision and split skin graft
G : Along Langer's lines
H : Split skin graft only
I : Microvascular free flap
1 : Posterior spinal instrumentation
Correct answer:
A
Your answer:
A
2 : Anterior cervical spine approach
Correct answer:
G
Your answer:
G
3 : Dupuytren's fasciectomy
Correct answer:
B
Your answer:
F
4 : Fasciotomy for lower leg compartment syndrome
Correct answer:
A
Your answer:
A

References

Question 122
Match the fracture description with the most appropriate classification
A : Bado Type 1
B : Bado Type 2
C : Bado Type 3
D : Bado Type 4
E : Galeazzi
F : Nightstick
G : Terrible triad
H : Bipolar fracture dislocation
I : Essex-Lopresti lesion
1 : Radial head dislocation with proximal third radius and ulna fractures.
Correct answer:
D
Your answer:
I
2 : Isolated midshaft ulna fracture
Correct answer:
F
Your answer:
F
3 : Proximal third ulna fracture with posterior radial head dislocation.
Correct answer:
B
Your answer:
B

Question 123
Which of the following structures are most at risk from these standard wrist
arthroscopic portals?
A : Deep branch of the radial artery
B : Superficial branch of the radial nerve
C : Extensor pollicis longus
D : Dorsal cutaneous branch of ulnar nerve
E : Palmar cutaneous branch of ulnar nerve
F : Ulnar artery
G : Lateral antebranchial cutaneous nerve
H : Posterior interosseous nerve
I : Radial collateral ligament
J : Ulnar collateral ligament
1 : The 6U radiocarpal portal
Correct answer:
D
Your answer:
D
2 : The 1-2 radiocarpal portal
Correct answer:
B
Your answer:
B
3 : The proximal radioulnar portal
Correct answer:
H
Your answer:
F

References

Wrist arthroscopy: Principles and Applications J Am Acad Orthop Surg 2001;9:200-9 Gupta et
al

Question 124
Concerning human respiratory physiology
A : compliance
B : functional residual capacity
C : vital capacity
D : tidal volume
E : dead space
F : shunt
G : minute volume
H : West's zones
I : FEV1
1 : Volume change per unit of distending pressure
Correct answer:
A
Your answer:
A
2 : The sum of residual volume and expiratory reserve volume
Correct answer:
B
Your answer:
B
3 : Theoretical areas of lung tissue which demonstrate varying effects of gravity on the
regional alveoli and corresponding pulmonary circulation
Correct answer:
H
Your answer:
H

References
Textbook of Anaesthesia. Aitkenhead and Smith. 3rd Edition pp1-12
West JB Respiratory Physiology - the essentials 4 ed. Williams & Wilkins, 1990. ISBN 0-683-
08942-0.
Nunn JF Nunn's Applied Respiratory Physiology 4 ed. Butterworth-Heinemann 1993. ISBN 0
7506 1336 X.
There are remarkable inequalities in blood flow through various regions of the lung. These
variations in flow are mainly related to height above the heart, which is not too remarkable
when one considers the low pressures present in the pulmonary circulation - hydrostatic
pressure alone will result in a pressure difference of approximately 30cm H20 from the base
of the lung to the apex. This led West to propose his "Three Zones" of the lung [West et al. J
Appl Physiol 1964 19 p713] - in the topmost Zone 1, there is no flow because the pressure is
not sufficient, in the lowest Zone 3 flow is continuous because intravascular pressure always
exceeds alveolar pressure, while in the middle zone 2, flow depends remarkably on the
difference between arterial and alveolar pressures - venous pressure is irrelevant as it is
lower than alveolar pressure. Note that in the normal lung, there should be NO zone 1, as the
pressure is just sufficient to perfuse the apices! Some have subsequently proposed a "zone
4" which might occur in the bases where poorly expanded lung might actually result in
narrowing of extra-alveolar vessels with reduced flow.

Question 125
Which is the single most likely diagnosis in each of the following scenarios?
A : C8/T1 root avulsion
B : Medial cord injury
C : High ulnar nerve injury
D : Low ulnar nerve injury
E : Radial nerve injury at elbow
F : Posterior cord injury
G : Lateral cord injury
H : High median nerve injury
I : Low median nerve injury (at wrist)
J : Axillary nerve lesion
1 : Loss of power of elbow flexion
Correct answer:
G
Your answer:
G
2 : Loss of thumb IPJ flexion but preservation of FDS power to ring and little fingers
Correct answer:
H
Your answer:
E
3 : Weakness of all hand intrinsic muscles with ipsilateral Horner’s syndrome
Correct answer:
A
Your answer:
A

References

Question 126
Match the following methods of fracture management with the following types of bone
healing
A : periosteal bridging callus and enchondral ossification
B : primary cortical healing (cutting cone type remodelling)
C : hypertrophic non-union
D : atrophic non-union
E : oligotrophic non-union
F : intramembranous ossification
G : distraction osteogenesis
H : enchondral ossification
I : none of above
J : infective non-union
1 : Cast immobilisation for closed fracture
Correct answer:
A
Your answer:
A
2 : Compression plate
Correct answer:
B
Your answer:
B
3 : Inadequate immobilisation and inadequate blood supply
Correct answer:
D
Your answer:
D

References

Review of Orthopaedics, Miller (4th Edition, p18)

Question 127
With regards to atraumatic osteonecrosis
A : Femoral head
B : Caisson disease
C : Gaucher's disease
D : Medial femoral condyle
E : Humeral head
F : Perthes' disease
G : Lunate
H : Capitellum
I : Sickle cell disease
J : Talus
1 : Which is the commonest site of occurrence
Correct answer:
A
Your answer:
D
2 : Kienbock’s disease involves which bone?
Correct answer:
G
Your answer:
G
3 : Which risk factor for osteonecrosis is characterised by lipid-laden macrophages?
Correct answer:
C
Your answer:
C

References
Apley's System of Orthopaedics and Fractures. Solomon, Warwick,Nayagam. 8th Ed 2001.
Arnold
Review of Orthopaedics.5th Edition. Editors Miller MD, Hart JA. 2008.Saunders Elsevier.
Gaucher's disease is caused by a deficiency of the enzyme glucocerebrosidase, leading to an
accumulation of its substrate, the fatty substance glucocerebroside, which is a cell membrane
constituent of red and white blood cells.

Question 128
With regards to hand anatomy
A : Dorsal interosseous muscles
B : Opponens pollicis
C : 2nd and 3rd lumbricals
D : 1st and 2nd lumbricals
E : Abductor pollicis brevis
F : 4th and 5th lumbricals
G : Adductor pollicis
H : Extensor pollicis brevis
I : Flexor pollicis brevis
J : Palmar interosseous muscles
1 : The ulnar nerve is the sole supply to which thenar muscle
Correct answer:
G
Your answer:
G
2 : The single muscle in the hand to have dual nerve supply is
Correct answer:
I
Your answer:
E
3 : Which of the intrinsic muscles of the hand other than thenar muscles are supplied
by the median nerve?
Correct answer:
D
Your answer:
D

References

Campbell's Operative Orthopaedics


In the forearm, the ulnar nerve supplies flexor carpi ulnaris, the ulnar half of flexor digitorum
profundus, abductor digiti minimi, opponens digiti minimi, palmaris brevis, flexor digiti minimi,
the 3rd and 4th lumbricals, interossei, adductor pollicis and the deep head of flexor pollicis
brevis.
The median nerve supplies the pronator teres, flexor carpi radialis, palmaris longus, flexor
digitorum superficialis, abductor pollicis brevis, the supinator head of flexor pollicis brevis,
opponens pollicis, 1st and 2nd lumbricals. The anterior interosseus nerve supplies the lateral
half of flexor digitroum profundus, flexor pollicis longus and pronator quadratus.
Soft Tissue and Joint Infections. In the following clinical situations, which is the most
likely pathogen?
A : Neisseria Gonorrhoea
B : Group A Strep
C : Coagulase Negative Staphylococcus
D : Staphylococcus Aureus
E : Salmonella
F : Pneumocystis Carinii
G : Pseudomonas
H : Neisseria Meningitidis
I : Haemophilus Influenza B
J : Methicillin Resistant Staph. Aureus
1 : A 75 year old with low grade peri prosthetic infection.
Correct answer:
C
Your answer:
C
2 : Osteomyelitis in an 18 year old with Sickle cell disease.
Correct answer:
D
Your answer:
D
3 : Septic Arthritis in a neonate.
Correct answer:
D
Your answer:
B

Question 130
Which is the most likely diagnosis with these findings?
A : Osteoid osteoma
B : Haemangioma
C : Chordoma
D : Chondrosarcoma
E : Osteoblastoma
F : Lymphoma
G : Giant cell tumour
H : Admantinoma
I : Myeloma
J : Eosinophilic granuloma
1 : A seventy year old man presenting with bone pain and punched out lytic bone
lesions on x-ray. Histology shows 'clock face' chromatin
Correct answer:
I
Your answer:
I
2 : A 67 year old lady presenting with neurological compromise and, a lytic sacral
defect on x-ray. Histology shows large vacuolated (physaliferous) cells in strands in a
mucus mass on histology
Correct answer:
C
Your answer:
C
3 : An 8 year old child with thoracic back pain and vertebral body flattening (vertebra
plana) on plain x-ray
Correct answer:
J
Your answer:
J

References

Multiple myeloma typically occurs in patients between 50 and 80 years old. The classic xray
appearance is of punched out lytic lesions, and histologically the classic appearance is sheets
of plasma cells. Well differentiated plasma cells have an eccentric nucleus and a peripherally
clumped chromatic “clock face”.
Chordoma is a malignant neoplasm is which the cell of origin derives from primitive
notochordal tissue. This lesion occurs predominantly at the ends of the vertebral column
(spheno-occipital and sacrum). There may be nerve compression. Sacral destruction will
often only be seen at a late stage on plain xray due to difficulties evaluating this area on plain
films. On histology the chordoma cells sometimes have a vacuolated appearance and are
called physaliferous cells. The chordoma cells are often in strands in a mass of mucus.
Eosinophilic granuloma is usually seen in children under 10 years of age, most often in the
thoracic spine. Classically causes vertebral flattening (vertebra plana) which is seen on lateral
films.
Miller 4th Edition

Question 131
With regards to steroid therapy
A : Modification of nuclear gene transcription
B : Change in cellular protein synthesis
C : Enhanced monocyte / macrophage function
D : Increased incidence of infection
E : Muscle atrophy
F : Adrenal insufficiency
G : Alterations in serum glucose levels
H : Avascular necrosis of bone
I : Peptic ulceration
J : Pancreatitis
K : Osteoporosis
L : Psychological disturbance
1 : All of the following effects are associated with glucocorticoid usage EXCEPT:
Correct answer:
C
Your answer:
C
2 : Which of the following effects of steroid usage necessitate gradual withdrawal upon
cessation?
Correct answer:
F
Your answer:
F
3 : Steroid-induced fatty liver change with subsequent showers of fatty emboli has
been cited as a possible cause of this complication
Correct answer:
H
Your answer:
H

References

Cruess RL, Ross D, Crawshaw E. The etiology of steroid-induced avascular necrosis of bone.
A laboratory and clinical study. Clin Orthop Relat Res. 1975 Nov-Dec;(113):178-83.
A study of human material showed intravascular fat and diminished number of osteocytes in
patients on systemic steroids who had died without clinical evidence of avascular necrosis. In
addition, intravascular fat was demonstrated in avascular femoral heads and humeral heads
both from transplant patients and patients receiving steroids for other reasons. The most
tenable explanation for this series of events involves steroid-induced fatty liver with
subsequent showeres of fatty emboli, which lodge in the subchondral region owing to the
microvascular anatomy. Cell death ensues, and the necrotic bone is partially removed by the
normal resorptive mechanism, but steroids retard osteogenesis, lead to microfractures, and
eventually sequester the involved area.

Question 132
Select the most appropriate structure that primarily performs the following function
A : Coracohumeral ligament
B : Glenoid labrum
C : Superior glenohumeral capsule
D : Middle glenohumeral ligament
E : Inferior glenohumeral ligament complex
F : Long head of biceps
G : Conoid ligament
H : Trapezoid ligament
I : Acromioclavicular ligament
J : Coracoacromial ligament
1 : Prevents anterior-posterior translation of the distal clavicle.
Correct answer:
I
Your answer:
I
2 : Stabilises anterior and inferior instability in abduction.
Correct answer:
E
Your answer:
A
3 : Together with the trapezoid ligament forms the coracoclavicular ligament
Correct answer:
G
Your answer:
G

References

The glenoid labrum deepens the socket and anchors the inferior glenohumeral ligament
complex.
The glenohumeral (GH)_ ligaments, including the coracohumeral, superior, middle and
interior glenohumeral ligaments limit excessive humeral head rotation or translation.
The superiorglenohumeral and coraco humeral ligaments limit inferior translation and external
rotation when the arm is adducted, and posterior translation when the arm is forward flexed,
adducted and internally rotated.
The middle glenohumeral ligament limits external rotation of the adducted humerus, inferior
translation of the adducted and externally rotated humerus, and anterior and posterior
translation of the partly adducted and externally rotated arm.
The inferior glenohumeral ligament is the primary restraint to anterior, posterior and interior
GH translation for 45 to 90 degrees of GH elevation.
The conoid and trapezoid ligaments are the coracoclavicular ligaments. They prevent inferior
translation of the coracoid and the acromion from the clavicle.
The acromioclavicular ligament resists AP translation of the clavicle and scapula.
Miller's review of orthopaedics..

Question 133
With regards to statistical tests
A : Chi-square test
B : Fisher's exact test
C : Student's T Test
D : ANOVA
E : Sensitivity
F : Specificity
G : Negative predictive value
H : Positive predictive value
I : P value
1 : The ability of a test to detect truly positive results
Correct answer:
E
Your answer:
E
2 : The most appropriate test for continuous data with a small sample size
Correct answer:
B
Your answer:
A
3 : The measurement for the probability of committing a type one error
Correct answer:
I
Your answer:
I

References

Chi Square Test concerns the frequency with which an observation occurs and is used for
qualtitative or non continuous discrete data.
Fisher’s exact test is used for smaller study numbers and produces a more conservative
estimate of the P value.
Student’s T test is used for continuous data.
ANOVA, the analysis of variance, is an extension of the t-test and relies on variation within a
group and between groups.
Sensitivity is the ability of a test to detect that which is truly positive.
Specificity is the ability of a test to detect that which is truly negative.
Positive predictive value is the chance that a condition is present if identified by a positive
test.
Negative predictive value is the opposite of positive predictive value, the chance that a
condition is not present as identified by a negative test result.
P value is the probability of committing a type 1 error. A type 1 error is rejecting a true null
hyposthesis, ie. a false positive result. The probability is reduced the smaller the p value, a p
value less than 0.05 is generally considered statistically significant. A type 2 error is accepting
a false null hypothesis, ie. a false negative result.

Question 134
With regard to developmental dysplasia of the hip, select the most appropriate first line
of management for the following
A : Gallows traction
B : Physiotherapy
C : Pavlik harness/ Abduction splint
D : Hydrotherapy
E : Ultrasound scan
F : Skin traction
G : Open reduction
H : Observation and/ or discharge
I : Pelvic osteotomy
J : Frog leg lateral x-ray
1 : 3 week old baby with clicky hip
Correct answer:
E
Your answer:
E
2 : 2 month old with DDH
Correct answer:
C
Your answer:
C
3 : Graf type 1 on USS
Correct answer:
H
Your answer:
C

References

Miller – 5th Edition. Pg 225-228


Dynamic ultrasonography is useful for making the diagnosis in young children before the
ossification of femoral head, which occurs at 4-6 months.
Hips can be classified into various categories (Graf types) depending on the alpha angle. Graf
type 1 hip means an alpha angle of more than 60o, which is normal. These children can be
discharged.

The most appropriate treatment for a dysplastic but enlocated hip at 2 months is some form of
abduction splint with Pavlik harness being the preferred choice of many.

Question 135
Please select the most appropriate response for the following statements
A : 24 hours high dose intravenous antibiotics
B : immediate incision and drainage of the hip
C : incision and drainage pending MRI scan next day
D : correct ankle equinus
E : correct forefoot adduction
F : correct 1st ray pronation
G : mobilise non-weight bearing and review at six weeks
H : in situ pinning with partially threaded 6.5mm screw at 90 degrees to slip
I : immediate Southwick osteotomy
1 : A 12 year old developed pain in his left knee 2 weeks ago. He is still able to weight
bear but frog lateral x-rays show a Grade II slip of his left capital femoral epiphysis.
What is the most appropriate management?
Correct answer:
H
Your answer:
H
2 : A 2 year old presents at 11 pm with pain and inability to use her right leg. She has
been systemically unwell with a temperature of 37.9 degrees. On examination the right
hip is held in flexion with extremely limited active or passive movements. Knee
examination is normal, as are the plain radiographs of the affected limb. WCC is
elevated along with CRP and ESR. What is the next line of management?
Correct answer:
B
Your answer:
B
3 : You are about to apply the first plaster for a baby with CTEV. What is the initial
manipulation aiming to achieve?
Correct answer:
F
Your answer:
F

References

OKU – 9th Edition Pg 715-722; Miller – 5th Edition Pg 233-237

Recommended treatment for Grade I and II SCFE is in-situ pinning. For severe grades of slip
controversy exists as to the role of primary open reduction with subcapital osteotomy or that
for corrective intertrochanteric osteotomy.
Because pus is chondrolytic, septic arthritis in children is a surgical emergency.
Ponseti casting begins with correction of cavus by aligning the first ray with the remaining
metatarsals. Subsequent manipulation aims to correct the forefoot adduction and heel varus.
Residual equinus requires Achilles tendon release in 90%.

Question 136
In shoulder
A : Bankart lesion
B : Long head of biceps
C : Rotational osteotomy
D : Subscapularis
E : HAGL lesion
F : Anterior capsulorrhaphy
G : SLAP lesion
H : Short head of biceps
I : Coracoid transfer
J : Hill Sachs lesion
1 : Which structure is advanced in a Putti-Platt procedure?
Correct answer:
D
Your answer:
D
2 : An inferior glenoid labral lesion is also known as
Correct answer:
A
Your answer:
A
3 : A significant Hill Sachs lesion is best treated by which procedure?
Correct answer:
C
Your answer:
C

References

Campbell's Operative Orthopaedics


Canale TS, Beaty JH

Oxford Textbook of Orthopaedics

Question 137
Regarding treatment of patella subluxation and dislocation
A : Patella alta
B : Patella hypermobility
C : Insall-Salvati index > 1.8
D : Lateral patella tilt and Q-angle >20 degrees
E : Patella baja
F : Hypoplastic trochlea
G : Lateral patellar tilt and normal Q-angle
H : Q-angle >20 degrees
I : Acute patellar dislocation with osteochondral fracture
J : Tight medial structures
K : Grade IV patella chondomalacia
L : Strongly positive patellar apprehension test
M : Tender medial retinaculum
N : Dislocation with tense haemarthrosis
1 : Which finding would be the most appropriate indication for arthroscopic lateral
release?
Correct answer:
G
Your answer:
G
2 : Which finding would be the most appropriate indication for a distal extensor
realignment procedure?
Correct answer:
H
Your answer:
H
3 : Which finding would be the most appropriate indication for open medial
retinaculum and vastus medialis repair?
Correct answer:
I
Your answer:
B

References

CAMPBELLS - SPORTS MEDICINE VOL iii - TENTH

Question 138
What is true for each of these skin preparation agents?
A : Acts by denaturing proteins
B : Acts by activation of free radicals
C : Effective only against spore forming organisms
D : Is a quaternary zinc compound
E : Has a 48 hour duration of action
F : Bactericidal but does not kill spore forming organisms
G : Rapidly inactivated by organic material such as blood
H : Creates ion channels in the bacterial cell wall
I : Is always inert on the skin
J : More effective against gram-negative organisms
1 : 70% Isopropyl alcohol
Correct answer:
A
Your answer:
A
2 : 0.5% Chlorhexidine
Correct answer:
F
Your answer:
E
3 : 70% Povidone - iodine
Correct answer:
G
Your answer:
G

References

http://www.surgical-tutor.org.uk/default-home.htm?principles/microbiology/asepsis.htm~right
Accessed on 9/12/2007
· 70% Isopropyl alcohol
o Acts by denaturing proteins
o Is bactericidal but short acting
o Effective against gram-positive and gram-negative organisms
o Also fungicidal and virucidal
· 0.5% Chlorhexidine
o Quaternary ammonium compound
o Acts by disrupting the bacterial cell wall
o Bactericidal but does not kill spore forming organisms
o It is persistent and has a long duration of action (up to 6 hours)
o More effective against gram-positive organisms
· 70% Povidone - iodine
o Acts by oxidation / substitution of free iodine
o Bactericidal and active against spore forming organisms
o Effective against both gram-positive and gram-negative organisms
o Rapidly inactivated by organic material such as blood
o Patient skin sensitivity is occasionally a problem
o No evidence that one is superior to any other

Question 139
In bleeding disorders
A : Rhesus incompatibility
B : Haemophilic arthropathy
C : Pigmented villonodular synovitis
D : Osteomyelitis
E : Christmas disease
F : Sickle cell disease
G : Deep venous thrombosis
H : Aspirin
I : Warfarin
J : Heparin
K : Haemophilia type A
L : Gilbert Syndrome
1 : widening of the intercondylar notch is a characteristic xray finding in this disease
Correct answer:
B
Your answer:
C
2 : potentiates antithrombin III
Correct answer:
J
Your answer:
J
3 : the aetiology of which is Factor IX deficiency
Correct answer:
E
Your answer:
E

References

Radiology Review Manual 5th Ed. Dahnert W. Lippincott Williams


Miller Review of orthopaedics, 4th edition p160.
Haemophilia A is deficiency of Factor VIII and Haemophilia B (Christmas disease) is
deficiency of Factor IX.
Haemophiliac arthropathy is caused by repeated bleeding into joints with resultant pannus
formation and synovitis which erodes cartilage with loss of subchonral bone plate and
formation of subarticular cysts. The knee is the commonest joint affected. Classic radiological
findings include “squaring” of the patella, widening of the intercondylar notch, and flattening of
the condylar surface. These changes are also seen in JIA. Treatment of the skeletal
manifestations involves synovectomy and total joint arthroplasty.
Heparin is a sulphated polysaccharide. It produces its major anticoagulant effect by
inactivating thrombin and activated factor X (factor Xa) through an antithrombin (AT)-
dependent mechanism. By inactivating thrombin, heparin not only prevents fibrin formation
but also inhibits thrombin-induced activation of platelets and of factors V and VIII.

Question 140
Which is the best tendon transfer option to address each of the following
A : Suture flexor digitorum profundus(FDP)side to side.
B : Extensor indicis proprius (EIP) to abductor pollicis brevis (APB) and flexor carpi radialis
(FCR) to flexor carpi ulnaris(FCU)
C : Radial half extensor indicis proprius (EIP) to to first dorsal interosseous and
brachioradialis to adductor pollicis.
D : Flexor carpi radialis (FCR) to abductor pollicis brevis (APB)
E : Palmaris longus (PL) to Extensor pollicis longus (EPL)
F : Extensor carpi radialis longus (ECRL) to extensor pollicis longus(EPL)
G : Flexor carpi radialis (FCR) to extensor carpi radialis (ECU)
H : Extensor indicis proprius (EIP) to abductor pollicis brevis (APB)and extensor carpi radialis
longus (ECRL) to flexor pollicis longus (FPL)
I : Palmaris longus (PL) to extensor pollicis brevis (EPB) and extensor carpi radialis longus
(ECRL) to flexor pollicis longus (FPL)
J : None of the above
1 : 1. Ongoing loss of thumb function 9 months after a radial diaphysis fracture.
Correct answer:
E
Your answer:
E
2 : 2. Restoration of index and middle finger function following a compression
neuropathy at the process of Struthers.
Correct answer:
A
Your answer:
J
3 : 3. Restoration of pinch grip using index and thumb finger function following a
compression neuropathy at the process of Struthers.
Correct answer:
C
Your answer:
H

References

In high radial nerve palsy there is loss of extensor function at wrist and thumb also loss of
abductor function of thumb. Functionally thumb restoration needs extension. This is achieved
by transfer of PL to EPL. Any one of FCR, FCU, FDS 3 could be used to restore MCPJ
extension by transfer to EDC and wrist extension by PT to ECRL/ECRB. In high median nerve
palsy there is loss of pronation, radial deviation of wrist with loss of finger flexion index and
middle finger and loss of flexion and opposition of thumb. To restore finger flexion ulnar half of
FDP is sutured side to side with median innervated radial half. In low ulnar nerve palsy, digital
abduction, adduction and adduction of thumb are lost. This leads to pinch grip weakness. To
restore pinch grip abduction of index finger is restored by powering first dorsal interosseous
by radial half of EIP and adduction of thumb by BR using tendon graft to go across palm
through 3rd interroseous space.
Basic orthopaedic sciences The Stanmore guide. Hodder Arnold 2007;208.
Campbell’s operative orthopaedics 10th Ed.
Trumble TE, Cornwall R, Budoff JE (eds). Core Knowledge in Orthopaedics: Hand, Shoulder
& Elbow. Elsevier, Philadelphia, 2005

Question 141
Which is the most likely cause for hyponatraemia in the following patients?
A : MDMA ingestion
B : SIADH
C : Thiazide diuretic usage.
D : Fluid overload
E : Dehydration.
F : Small bowel obstruction.
G : Nephrotic syndrome.
H : Vigorous exercise.
I : Primary Aldosteronism
J : Addison's Disease.
1 : A 17 year old male suddenly severely ill, brought in by his friends at 0230hrs, with
lethargy, confusion, hypotension and acute hyponatraemia.
Correct answer:
A
Your answer:
A
2 : A 57 year old man with haemoptysis, hyper calcaemia and sub-trochanteric fracture
of the femur.
Correct answer:
B
Your answer:
B
3 : A 78 year old lady during post operative recovery from intramedullary nailing of
femoral fracture complains of shortness of breath, peripheral oedema and orthopnoea.
Correct answer:
D
Your answer:
D

References

http://www.emedicine.com/emerg/topic275.htm> ‘Hyponatraemia’ Craig S. 2008


MDMA-induced hyponatremia occurs via multiple mechanisms; these include the induction of
syndrome of inappropriate antidiuretic hormone (SIADH), the encouragement to drink large
amounts of water to prevent unpleasant side effects of the drug, and the tendency among
those intoxicated to be involved in vigorous physical activity that results in heavy sweating.
SIADH is associated with malignant lung disease, which predisposes to metastatic
pathological fracture.

Question 142
For each of the following scenarios, select the most likely diagnosis or most
appropriate treatment
A : Eosinophilic Granuloma
B : Sequestrectomy and IV antibiotics
C : Chemotherapy and irradiation
D : Chondroblastoma
E : Osteomyelitis
F : Chemotherapy, surgical resection +/- irradiation
G : Local irradiation
H : Metastatic Lymphoma
I : Osteochondroma
J : Surgical resection only
K : Osteosarcoma
L : Giant Cell Tumour
1 : A 10-year-old boy presented with gradual onset of pain in the left knee. His mother
has noticed that sometimes he is pyrexial. Blood tests showed a high ESR and WCC
but a normal CRP. Radiographs showed a destructive lesion in the metaphysis of the
left distal femur. Treatment is:
Correct answer:
F
Your answer:
B
2 : A 12-year-old patient presented with a knee pain. Radiographs showed an
epiphyseal lesion. The most likely diagnosis is
Correct answer:
D
Your answer:
L
3 : A 27-year-old woman presented with increasing pain and swelling around the right
knee for 4 weeks. She denies any trauma. Examination revealed a painless nodular
lump in her neck. All blood tests were within normal limits. Radiographs showed a
metaphyseal lesion in the distal femur with mottled appearance. The most likely
diagnosis is
Correct answer:
H
Your answer:
H

References

Miller Review of orthopaedics, 4th edition p487.


Musculoskeletal Cancer Surgery Treatment of Sarcomas and Allied Diseases by Martin M.
Malawer and Paul H. Sugarbaker Kluwer Academic Publishers, 2001
Simon MA, Springfield DS (eds): Surgery for Bone and Soft Tissue Tumours. Philadelphia,
PA, Lippincott-Raven, 1998, pp190-191
For question 1 the diagnosis is Ewing's sarcoma for which the treatment is option F
Radiotherapy is used in cases where wide surgical margins are not achieved. Osteomyelitis is
unlikely as CRP is normal. For question 2, chondroblastoma affects mainly children and is a
well circumscribed lytic epiphyseal lesion. For question 3, the clues are the painful neck
nodule (lymph node), metaphyseal lesion and the mottled appearance on radiographs.
Lymphoma can present with normal blood tests.

Question 143
With regards to treatment of midfoot fracture - dislocation
A : Below knee walking plaster
B : Below knee non weight bearing plaster
C : Examination under anaesthesia
D : Closed reduction and plaster
E : Open reduction and fixation with screw only
F : Open reduction and fixation with screws and K-wires
G : Open reduction, bone graft and internal fixation
H : Open reduction and external fixation
I : Open reduction and primary fusion
1 : A 23 year old athletic man has a symptomatic tuberosity fracture of the navicular
after being managed for 3 months in plaster cast with no suggestion of union on CT
scan
Correct answer:
G
Your answer:
G
2 : A 43 year old road traffic accident casualty whose x-rays demonstrate malalignment
of the 2nd to 5th metatarsals at tarso metatarsal joints.
Correct answer:
F
Your answer:
I
3 : An 18 yr old woman presents after twisting injury to ankle. Her x-rays show a
normal ankle and a cortical avulsion of the cuboid without subluxation or dislocation.
Correct answer:
A
Your answer:
A

References

Bucholz R.W., Heckman J.D., Court-Brown C., Tornetta P., Koval K.J., Rockwood and
Green's Fractures in Adults; Lippencott, Williams and Wilkins, Philadelphia, 2005: 2182-2214

current user: tom quick (6031674)


 Operative Records
 Portfolio
 Training
 Admin & Help
 Security
 Logout
Test Validated
Exam:UKITE 2008
Diet:UKITE 2008 Severn Institute (Bristol)

The Examination has now been completed and validated

Your Final result

Total Number of Questions : 143


Maximum achievable Score : 208
Your Score : 129
Percentage : 62.02 %
Please provide feedback to help us improve the exam

172/207 = 83%

S-ar putea să vă placă și